MIX TOPICS - BARBRI MCQ Flashcards

1
Q

A husband was on trial for the murder of his wife. During the course of the trial, the prosecution sought to introduce evidence that, six months before the wife’s death, the husband had purchased a large insurance policy on her life, and two weeks prior to her death he purchased two more large policies on the wife’s life from separate insurance carriers. All notice requirements have been met.

If the defense objects to admission of such evidence, should the objection be sustained?

A. Yes, because such evidence would be more prejudicial than probative.

B. Yes, because evidence of insurance is inadmissible as a matter of public policy.

C. No, because the evidence tends to establish motive.

D. No, because the evidence tends to establish criminal propensity.

A

CORRECT ANSWER: C. No, because the evidence tends to establish motive.

Evidence of the husband’s purchases of insurance on the wife’s life shortly before her death is admissible because it has a tendency to make the husband’s murder of his wife more probable than it would be without the evidence. As a general rule, all relevant evidence is admissible if offered in an unobjectionable form or manner, as is the case here. This evidence is relevant because it establishes a motive for the murder, and facts showing motive for doing an act are circumstantial evidence that the act was done. Because there are no other grounds for excluding the evidence, it should be admitted. (A) is incorrect because exclusion of evidence on the ground of prejudice is a matter within the trial judge’s broad discretion, and Federal Rule 403 requires that the evidence’s probative value be substantially outweighed by the danger of unfair prejudice for it to be excluded. While all evidence is prejudicial to the adverse party, “unfair” prejudice refers to evidence that suggests a decision on an emotional or otherwise improper basis; evidence of the purchase of life insurance policies does not fall within this category. (B) is incorrect because it misapplies the rule excluding evidence of insurance. Federal Rule 411 excludes evidence of liability insurance on the issue of whether a person acted negligently or wrongfully. Evidence of insurance coverage (particularly where it is life insurance on the life of a homicide victim) is relevant and admissible for other purposes. (D) is incorrect for two reasons: First, the purchase of insurance does not establish a criminal disposition or a propensity to do criminal acts. Second, evidence of conduct offered to show criminal propensity is inadmissible character evidence under Federal Rule 404. Thus, if the evidence were offered to show criminal propensity, the objection should be sustained, not overruled. On the other hand, evidence of prior conduct (or misconduct) is specifically admissible under Rule 404(b) to show the defendant’s motive, which is what the insurance evidence is intended to show.

How well did you know this?
1
Not at all
2
3
4
5
Perfectly
2
Q

A witness is called in a contract action between a plaintiff and a defendant. The witness takes his oath and testifies. During cross-examination, the defendant’s attorney asked the witness this question: “Isn’t it true that even though you took an oath to tell the truth so help you God, you are an atheist and don’t even believe in God?”

Upon the proper objection, will the judge require that the witness answer this question?

A. Yes, because the question is relevant to the witness’s character for truthfulness.

B. Yes, because instead of taking the oath, the witness could have requested to testify by affirmation without any reference to God.

C. No, because evidence of the beliefs or opinions of a witness on matters of religion is not admissible to impair credibility.

D. No, because an attack on the competency of a witness must be made at the time the witness is sworn.

A

CORRECT: C. No, because evidence of the beliefs or opinions of a witness on matters of religion is not admissible to impair credibility.

The judge should not require that the witness answer the question because evidence of the religious beliefs of a witness is not admissible to challenge credibility. Lack of religious belief is no longer a basis for excluding a witness. Not only are a person’s religious convictions irrelevant in determining the competence of a witness, Federal Rule 610 provides that a witness’s religious beliefs or opinions are not admissible to show that the witness’s credibility is thereby impaired or enhanced. Thus, (C) is correct and (A) is incorrect. (B) is incorrect. While it is true that the witness could have requested a different type of oath, Rule 610 prohibits this type of question because it would have shown his lack of religious beliefs. (D) is incorrect because, as discussed above, lack of religious belief is no longer a basis for disqualification; thus, this would not constitute an attack on the witness’s competency.

How well did you know this?
1
Not at all
2
3
4
5
Perfectly
3
Q

A town in a rural state facing financial difficulties passed a variety of “sin taxes,” including one aimed at electronic game arcades frequented by local juveniles. The tax is a one cent per game tax imposed on the manufacturers of the games based on the estimated number of plays over a machine’s lifetime. There are no electronic game manufacturers in the state.

Which of the following constitutional provisions would support the best argument against enforcement of the tax?

A. The Equal Protection Clause.

B. Substantive due process.

C. The Privileges and Immunities Clause of Article IV.

D. The Commerce Clause.

A

CORRECT ANSWER: D. The Commerce Clause

The best argument against enforcement of the tax is that it violates the Commerce Clause. If Congress has not adopted laws regarding a subject, local governments are free to tax or regulate local aspects of the subject area as long as the tax or regulation does not discriminate against interstate commerce or unduly burden it. Here, the tax does not discriminate against interstate commerce, since it does not single out interstate commerce for taxation in order to benefit the local economy. However, it could be argued that the tax unduly burdens interstate commerce. A local tax will be held to unduly burden interstate commerce if the locality’s need for the revenue does not outweigh the burden on interstate commerce. The Supreme Court will consider whether there is a substantial nexus between the activity or property taxed and the taxing state, whether the tax is fairly apportioned, and whether there is a fair relationship between the tax and the benefit the taxed party receives from the state. Here, there is little nexus between the manufacturer and the town. The facts indicate that out-of-state manufacturers’ machines are used in the town, but do not indicate whether the manufacturers conduct any selling activity in the town. Similarly, nothing indicates that there is a relationship between the tax and any benefit that the manufacturers derive from the town. Thus, the tax would probably be unconstitutional under the Commerce Clause. (A) is not as good an argument as (D) because the Equal Protection Clause prohibits the states from treating similarly situated persons differently without sufficient justification. Where a classification does not involve a suspect or quasi-suspect class or a fundamental right, the classification will be upheld as long as it is rationally related to a legitimate government interest. While the tax here singles out arcade game manufacturers for special tax treatment, no suspect or quasi-suspect class is involved, nor is a fundamental right affected. Thus, the tax will be valid under the Equal Protection Clause because it is rationally related to the legitimate government interest of raising revenue. (B) is not a good argument because substantive due process requires that laws not be arbitrary. When laws do not involve a fundamental right, they will be held valid under the Due Process Clause as long as they are rationally related to a legitimate government interest. As established above, no fundamental right is involved and the tax is rationally related to a legitimate government interest. Thus, under the Due Process Clause the tax may be enforced. (C) is not a good argument because the Privileges and Immunities Clause of Article IV prohibits states from discriminating against out-of-state residents when a fundamental right is involved, and the tax here does not differentiate between residents and nonresidents.

How well did you know this?
1
Not at all
2
3
4
5
Perfectly
4
Q

The defendant is charged with the battery of a bouncer at a local tavern. At the trial, the prosecutor introduces evidence that while the bouncer was attempting to question the defendant about her intoxicated demeanor, the defendant committed a battery on the bouncer. The defendant attempts to defend against the charge on the basis of self-defense, insisting that the bouncer used excessive force in stopping her from entering the tavern. The defendant attempts to introduce into evidence an authenticated copy of the tavern records that show that three patrons had written complaints against the bouncer within the past six months for the use of excessive force. The prosecutor objects on the grounds that the records are inadmissible character evidence.

Should the court sustain the objection?

A. Yes, because the character of a victim can be established only by reputation or opinion evidence.

B. Yes, because there is no evidence that the incidents involving the three patrons were based on the same facts as the defendant’s claim.

C. No, because the records were authenticated.

D. No, because the character trait of a victim may be established by opinion evidence, reputation evidence, or specific acts of misconduct.

A

CORRECT ANSWER: A. Yes, because the character of a victim can be established only by reputation or opinion evidence.

The court should sustain the objection because the records are evidence of specific bad acts. The Federal Rules permit a defendant to introduce evidence of a bad character trait of the alleged victim if it is relevant to the charge or the defense, but limit it to reputation and opinion evidence. Evidence of specific acts of the person in question that demonstrates that person’s character is permitted only in a few instances, such as if the acts are relevant to some issue other than disposition to commit the crime charged. Here, no issue is raised by this evidence other than the bouncer’s propensity to use excessive force. (A) is therefore correct and (D) is wrong. (B) is wrong because the facts do not have to be identical if evidence of bad acts were otherwise admissible. (C) is wrong; documentary evidence, even if fully authenticated and relevant, may be excluded if it violates a rule of competency, such as the rule for character evidence. Here, the objection should be sustained because the document is improper evidence of a specific bad act.

How well did you know this?
1
Not at all
2
3
4
5
Perfectly
5
Q

On February 1, the owner of a bowling alley read in a magazine an ad from a major manufacturer of bowling balls offering sets of 40 balls in various weights and drilled in various sizes for $10 per ball. The owner immediately filled out the order form included in the ad for the 40 balls and deposited it, properly stamped and addressed, into the mail. On February 2, the bowling alley owner received in the mail a letter from the manufacturer, sent out as part of its advertising campaign, stating in relevant part that it will sell the bowling alley owner 40 bowling balls at $10 per ball. A day later, on February 3, the manufacturer received the bowling alley owner’s order. On February 4, the balls were shipped.

On what day did an enforceable contract arise?

A. February 1, the day the bowling alley owner deposited his order in the mail.

B. February 2, the day the bowling alley owner received the letter from the manufacturer.

C. February 3, the day the manufacturer received the bowling alley owner’s letter.

D. February 4, the day the balls were shipped.

A

CORRECT ANSWER: D. February 4, the day the balls were shipped.

The contract arose when the balls were shipped. The general rule is that an offer can be accepted by performance or a promise to perform unless the offer clearly limits the method of acceptance. Here, the offer would be the bowling alley owner’s order, because a magazine ad is usually held to be merely solicitation to accept offers rather than an offer. Thus, the manufacturer accepted and the contract was formed when it shipped the balls. (A) is wrong because the bowling alley owner’s order was an offer to buy, and no contract could be formed until that offer was accepted. (B) is wrong because this is a case of crossing offers; even though both offers contain the same terms, they do not form a contract. (C) is wrong because no contract will be formed until there has been an acceptance, and, as stated, the bowling alley owner’s letter was merely an offer.

How well did you know this?
1
Not at all
2
3
4
5
Perfectly
6
Q

A store sold office equipment and supplies to various businesses in the area. The store entered into a written agreement with an electronics company to purchase all of its monthly requirements of printers for a period of five years at a specified unit price. The agreement contained a clause prohibiting assignment of the contract. Shortly thereafter, the electronics company assigned the contract to a finance company as security for a loan. The store subsequently ordered the printers from the electronics company, which were delivered on time.

Which of the following accurately states the legal effect of the clause prohibiting assignment of the contract?

A. The clause as properly interpreted was not breached, and the assignment was effective.

B. The clause made the assignment to the finance company ineffective.

C. The electronics company’s assignment was a breach but was nevertheless effective to transfer to the finance company the electronics company’s rights against the store.

D. The clause is effective if the parties can establish a rational reason for including the covenant into their agreement.

A

CORRECT ANSWER: A.

A is Correct. The clause was not breached, and the assignment was effective. Under the UCC, which governs this sale-of-goods case, “unless the circumstances indicate the contrary, a prohibition of assignment of ‘the contract’ is to be construed as barring only the delegation to the assignee of the assignor’s performance.” Here, the electronics company assigned to the finance company the right to receive payment on its contract with the store. There was no delegation of duties to the finance company (the assignee). Therefore, when the electronics company “assigned the contract” to the finance company, it assigned only the right to payments, and it did not breach its contract with the store. (B) is wrong because the only time a nonassignment clause makes an assignment ineffective is when the clause itself states that any assignment will be void or when the assignee has notice of the nonassignment clause. (C) is wrong because the assignment was not a breach. (C) is stating the rule for when the contract contains a clause prohibiting the assignment of “contractual rights.” In that case, the assignment is effective, but the obligor may sue for breach. Here, the contract prohibits assignment of the contract, not contractual rights. (D) is wrong because the covenant would not stand or fall on its rationale.

How well did you know this?
1
Not at all
2
3
4
5
Perfectly
7
Q

A town adopted an ordinance providing that a person must have been a resident of the town for at least one year to be eligible to vote in school board elections. A resident who moved to the town seven months ago attempted to register to vote in the school board elections scheduled for the next month. However, the town clerk refused to register the resident because he will not have resided in the town for a full year prior to the election. The resident filed a class action suit on behalf of all of the new residents of the town, challenging the validity of the one-year residency requirement.

Which of the following statements is correct?

A. If the resident’s suit is not heard before the election, it will be dismissed as moot, because the resident will have met the residency requirement by the time of the next annual election.

B. The resident will prevail even if the matter is not decided until after next month’s election

C. As long as there is some legitimate purpose for the one-year residency requirement, such as the need to prepare voting lists, the residency requirement will be upheld.

D. The resident will lose because one-year residency requirements have been held to be permissible restrictions on the right to vote in local elections.

A

CORRECT ANSWER: B. The resident will prevail even if the matter is not decided until after next month’s election

The resident will prevail even if the matter is not decided until after the election, because the suit is not moot and the residency requirement is unconstitutional. The resident’s suit is not moot even if the matter will not be decided until after the election because other members of the class might have a live controversy. Under the case and controversy requirement of the Constitution, there must be a real, live controversy at all stages of the suit. If through the passage of time, the controversy between the parties is resolved, the case is said to be moot. However, there are exceptions to the mootness doctrine. In a class action, it is not necessary that the suit by the named plaintiff be viable at all stages, as long as the claim is viable by some member of the class. Thus, the suit here would not be moot. Moreover, the residency requirement here violates the resident’s fundamental rights to vote and to interstate travel. A restriction on the right to vote is subject to strict scrutiny and is valid only if it is necessary to achieve a compelling state interest (otherwise the restriction violates the Equal Protection Clause by treating new residents differently from old residents). Relatively short residency requirements (e.g., 30 days) have been upheld as being necessary to promote the compelling interest of assuring that only bona fide residents vote. However, the Supreme Court has struck down longer durational requirements for lack of a compelling justification. Thus, the one-year requirement here probably unconstitutionally impinges on the right to vote. The residency requirement also impinges on the fundamental right to travel in the same manner (i.e., it discourages people from migrating by denying them the right to vote without a compelling reason). Thus, the requirement is invalid. (A) is incorrect because, as indicated above, the case will not be moot since other members of the class might have a live controversy. (C) is incorrect because it applies the wrong standard. Because fundamental rights are affected by the residency requirement here, the government must show a compelling justification; a mere rational or legitimate basis is not enough. (D) is incorrect because, as stated above, the Supreme Court has found that there was no compelling interest for a one-year residency requirement in order to vote.

How well did you know this?
1
Not at all
2
3
4
5
Perfectly
8
Q

A retailer entered into an oral contract with an office supply wholesaler to buy 100 file boxes for an upcoming back to school sale at the retailer‘s store. The wholesaler agreed to deliver the file boxes in two weeks at a cost of $4 per file box. A week later, the retailer phoned the wholesaler and asked if she could increase her order to 200 file boxes. The wholesaler agreed. The wholesaler delivered the 200 file boxes as promised, but the retailer accepted only 150 upon discovering that she lacked storage space for all 200.

May the wholesaler recover damages with respect to the 50 file boxes that were not accepted?

A. Yes, because the retailer accepted $600 worth of file boxes.

B. Yes, because the modification was for less than $500.

C. No, because the contract as modified was for $800.

D. No, because the wholesaler is a merchant with respect to file boxes.

A

CORRECT ANSWER: C. No, because the contract as modified was for $800.

The wholesaler may not recover damages. Under the UCC Statute of Frauds, a contract for the sale of goods for $500 or more is unenforceable unless evidenced by a writing signed by the party sought to be held liable. The original contract was for $400 and, thus, was not within the Statute. Whether a modification must be in writing to be enforceable depends on whether the entire contract price as modified is within the Statute. Here, the retailer and the wholesaler modified their original contract to 200 file boxes, bringing the total price to $800. Thus, the modification was unenforceable under the Statute of Frauds, and the wholesaler cannot collect damages with respect to the 50 unaccepted file boxes. (A) is incorrect. Acceptance is an exception to the Statute of Frauds—but only to the extent of the goods accepted. That is, an oral contract for the sale of goods for $500 or more is enforceable to the extent the goods are accepted. Here, the original contract for 100 boxes is enforceable, but the modification is not. However, since the retailer accepted 50 additional boxes, the modified contract is enforceable to the extent of the additional 50 boxes accepted. The fact that the accepted amount meets the Statute of Frauds $500 threshold does not make the contract enforceable for all 200. (B) is incorrect because, as noted above, when determining whether a contract for the sale of goods is enforceable, we look at the whole contract price as modified; the price of the modification itself does not matter. (D) is incorrect because the fact that the wholesaler is a merchant with respect to the goods being sold (file boxes) has no bearing on the enforceability of the contract here.

How well did you know this?
1
Not at all
2
3
4
5
Perfectly
9
Q

The owner of an old car parked it in front of his house with a “for sale” sign in the windshield. In response to an inquiry from his neighbor, the car owner said that he would take $400 for the car. The neighbor responded, “You’ve got a deal.” Because it was a Sunday, and the banks were closed, the neighbor told the car owner that he would come to his house with the $400 the next day at about 6 p.m. The car owner said that was fine. At 9:15 the next morning, the car owner called his neighbor and told him that when they had talked the previous day, he forgot that he had just put two new tires on that car and that he would need an extra $50 to cover their cost. The neighbor agreed to bring $450 in cash to the car owner’s house at about six o’clock.

Is the neighbor legally bound to pay the car owner the additional $50?

A. Yes, because the original contract was not in writing.

B. Yes, because the contract, as modified, does not need to be in writing.

C. No, because no additional consideration was given for the oral modification.

D. No, because neither the neighbor nor the car owner is a merchant.

A

CORRECT ANSWER: B. Yes, because the contract, as modified, does not need to be in writing.

The neighbor must pay the car owner the additional $50 because the parties have an enforceable contract. A contract for the sale of goods (the car) was formed when the neighbor said, “You’ve got a deal.” The parties then orally agreed to a modification of the contract when the car owner called his neighbor the next morning. Under the Statute of Frauds provision in the UCC, which applies to all contracts for the sale of goods, a promise requires a writing signed by the party to be charged to be enforceable if it is for the sale of goods of $500 or more. Here, the contract as modified is under $500, so it is enforceable even though it is not in writing. (A) is incorrect because the fact that the original contract was not in writing is irrelevant to the issue of whether the modified contract is enforceable. If the modification had caused the contract to reach or exceed $500, the car owner could not have collected the additional $50 from his neighbor. (C) is incorrect because under UCC section 2-209, no consideration is needed for a good faith modification of a contract for the sale of goods. (D) is incorrect because the UCC rules on modifications and the Statute of Frauds apply to all contracts for the sale of goods, not just those between merchants.

How well did you know this?
1
Not at all
2
3
4
5
Perfectly
10
Q

An owner of land who was also a home contractor agreed in writing with a buyer to build a house to the buyer’s specifications on his (the contractor’s) land and then sell the house and lot to the buyer. The contract provided that the house was to be completed by March 1, with full payment due at that time. On March 1, the house was nearly complete, but due to delays in the delivery of materials, the contractor would need 20 more days to finish construction. On March 5, after discovering that the house had not yet been completed, the buyer notified the contractor in writing of her election to cancel the contract because of the contractor’s failure to deliver the house by March 1. The contractor responded that, due to an unanticipated strike at his supplier’s company, performance had been unforeseeably delayed and that the house would be ready by March 20. The buyer responded that she would no longer accept delivery of the house and land. The contractor then brought an action to recover damages for breach of contract.

Who will prevail?

A. The buyer, because the express date listed in the contract indicates that time was of the essence.

B. The buyer, because delivery by March 1 was a condition precedent to the buyer’s performance.

C. The contractor, because the strike was truly an unforeseeable intervening event.

D. The contractor, because the buyer received the substantial benefit of the bargain.

A

CORRECT ASWER: D. The contractor, because the buyer received the substantial benefit of the bargain.

The contractor will prevail because the buyer received the substantial benefit of her bargain. The failure to perform on time is a breach of contract, but in this case, it was a minor breach. Unless the nature of the contract is such as to make performance on the exact day agreed upon of vital importance, or the contract provides that time is of the essence, failure to perform at the stated time is not a material breach. Here, the home was nearly complete, and the delay was relatively short. The contract did not specify that time was of the essence; thus, the breach was minor. The remedy for a minor breach is damages; the aggrieved party is not relieved of her duty to perform. (A) is incorrect because merely stating a date for performance does not indicate that time is of the essence. There must be some explicit statement indicating that time is of the essence. (B) is incorrect. Although the delivery on March 1 is a condition precedent to the buyer’s duty to pay, the condition is excused by substantial performance. The test for whether a party has substantially performed is the same as the one for assessing whether a breach is minor or material. Here the breach is minor, the contractor substantially performed, and the condition is excused. (C) is incorrect because an unforeseeable event does not discharge a party’s duty to perform. A strike at the contractor’s supplier does not rise to the level of impossibility or impracticability, which would discharge his duty to perform. The contractor could have procured the supplies elsewhere.

How well did you know this?
1
Not at all
2
3
4
5
Perfectly
11
Q

An electronics store located in Los Angeles, California, purchases electronics from an electronics corporation located in San Antonio, Texas. On January 19, the electronics store mailed to the corporation an order for $200,000 worth of electronics, specifying that delivery was to be no later than February 25, F.O.B. South Texas Railroad Depot, San Antonio, Texas.

The corporation delivered the electronics to the railroad depot on February 15 and notified the electronics store by fax that its order was scheduled to arrive at the Southern Pacific Railroad Depot in Los Angeles on February 20. On February 17, the corporation learned that the electronics store was insolvent. The corporation demands immediate payment in cash for the electronics.

Is this demand permissible?

A
No, because the corporation may only refuse to tender delivery of the goods by stopping them in transit.

B
No, because the contract terms specifying delivery F.O.B. at the South Texas Railroad Depot passed title to the goods to the electronics store when the goods were delivered to the South Texas Railroad Depot.

C
No, because the payment terms were expressly stated in the purchase order of January 19.

D
Yes, because the electronics store is insolvent.

A

CORRECT ANSWER: D. Yes, because the electronics store is insolvent.

The corporation’s demand is permissible. If a buyer is insolvent, UCC section 2-702 permits the seller to refuse to deliver except for cash, including payment for all goods previously delivered under the contract. It does not matter that title passed on delivery to the South Texas Railroad Depot. Thus, (D) is correct, and (B) is wrong. (A) is wrong because while the corporation could stop the goods in transit under UCC section 2-705, even after title passes, it could also demand cash payment. (C) is wrong because the UCC provisions allow the above modifications to payment terms upon finding the buyer insolvent.

How well did you know this?
1
Not at all
2
3
4
5
Perfectly
12
Q

A farmer orally agreed to lease a tractor from a company for $500. However, when the company’s secretary put the agreement into writing she accidentally typed in a charge of $300. Both the company and the farmer signed the contract without noticing the mistake in the price. When it came time for payment, the farmer refused to pay more than $300 for the rental of the tractor.

If the company brings an action for the difference between the payment as orally agreed and as memorialized in the contract, which of the following, if proven, would most benefit the company?

A
The parties made a mistake in integration.

B
The writing was only a partial integration.

C
The writing was intended as a sham.

D
The company or the farmer misunderstood the amount the rental payment was to be.

A

CORRECT ANSWER: A. The parties made a mistake in integration.

The most beneficial scenario is if the parties made a mistake in integration. Where the parties make a mistake in integrating their agreement into a writing, the courts will permit the error to be corrected. (B) is not helpful to the company because a writing that is a partial integration cannot be contradicted, it may only be supplemented by proving up consistent additional terms. (C) is wrong because if the writing were a sham, no rights would arise from it. (D) is wrong because if one of the parties made a mistake as to the payment amount, he would not necessarily be entitled to relief.

How well did you know this?
1
Not at all
2
3
4
5
Perfectly
13
Q

A woman decided to have a painting done of herself. She contracted in writing with an artist, who agreed to paint the woman for $10,000. The fee was payable on completion of the painting, provided that the painting was to the woman’s “complete and utter satisfaction.” On the same afternoon that the artist entered into the contract with the woman, he assigned the contract to his cousin. The artist then painted the woman’s picture. After the job was done, the woman told him, “That’s a very good likeness of me, but it shows my defects, so I’m not satisfied.” She refused to accept the painting or to pay the artist or his cousin.

Can the cousin recover from the woman?

A
Yes, because the condition in the agreement between the woman and the artist did not apply to his cousin.

B
Yes, because otherwise an unjust enrichment will occur.

C
No, because rights arising under personal services contracts are not assignable.

D
No, because the woman was not satisfied with the painting.

A

CORRECT ANSWER: D. No, because the woman was not satisfied with the painting.

D is Correct. The cousin will not recover from the woman because she has a defense inherent in the contract. When one of the original parties to a valid contract assigns his rights under the contract to a third party, the assignee may enforce his rights against the obligor directly but is generally subject to any defenses that the obligor had against the assignor. As long as the defense is inherent in the contract, such as failure of a condition, it is always available against an assignee because it was in existence when the contract was made (even if whether the obligor would be able to utilize it was uncertain). Here, the artist (the assignor) and the woman (the obligor) had a valid contract—her promise to purchase the painting only if she was satisfied with it is not illusory because she has to exercise her right of rejection in good faith. When the artist assigned his rights under the contract to his cousin (the assignee), his cousin became subject to the condition in the contract that the woman be satisfied with the painting. Her dissatisfaction with the painting excuses her duty to pay for it; this is a defense inherent in the contract that precludes the cousin’s recovering from the woman. (A) is incorrect because the assignee always takes subject to conditions in the original agreement between the obligor and the obligee. The only defenses that the obligor could not raise against the assignee are setoffs and counterclaims unrelated to the assigned contract that came into existence after the obligor learned that the contract was assigned. (B) is incorrect because the woman has not been enriched by the artist’s services. She has justifiably refused to accept the painting and has received no benefit from the transaction that would constitute unjust enrichment. (C) is incorrect because the only right that the artist has assigned is the right to receive payment from the woman if she accepted the painting. The woman’s duty is the same regardless of to whom she has to pay the money; therefore, the artist could validly assign his right to his cousin. Note that the analysis would be different if the artist had also attempted to delegate his duty of painting the woman to his cousin: duties involving personal judgment and skill may not be delegated. When an assignor assigns “the contract,” the words are interpreted as including a delegation of the duties unless a contrary intention appears. Here, the contrary intention is indicated by the fact that the artist did the painting rather than his cousin; hence, there was no attempt by the artist to delegate a nondelegable duty.

How well did you know this?
1
Not at all
2
3
4
5
Perfectly
14
Q

A homeowner entered into a written agreement with a contractor to remodel the homeowner’s basement for a set amount. Among the contract’s specifications was a provision that the contractor was to use an imported Brazilian hardwood for the paneling. The contractor began the job, and then assigned his right to payment under the agreement to a creditor. When the job was a little more than half completed, the homeowner sold the house to a buyer. In a document separate from the deed of conveyance, the homeowner assigned her right to the contractor’s labor and delegated her duty to pay under the contract to the buyer. The contractor completed the job on time but used Wyoming knotty pine instead of Brazilian hardwood for the paneling. The knotty pine is considered to be inferior to the Brazilian hardwood and is much less expensive, and the buyer was very displeased when he saw the result.

Who can the buyer sue for breach?

A
The original homeowner only.

B
The contractor only.

C
The contractor and his creditor.

D
The homeowner, the contractor, and the creditor.

A

CORRECT ANSWER. B. The contractor only.

B is Correct. The buyer can sue only the contractor for breach. The effect of a valid assignment of contract rights is to establish privity of contract between the obligor and the assignee while extinguishing privity between the obligor and assignor. The assignee then replaces the assignor as the real party in interest and he alone is entitled to performance under the contract. As the real party in interest, he may enforce his rights against the obligor directly. Here, the original homeowner (the assignor) has assigned to the buyer (the assignee) the right to receive the performance of the contract by the contractor (the obligor). Because the contractor has materially breached the contract by departing from the specifications and using a cheaper paneling, the buyer can sue the contractor for breach. (A) and (D) are incorrect because the buyer has no grounds for suing the original homeowner (the assignor). The assignor does not guarantee that the obligor will perform the contract for the assignee. The assignor only warrants that (i) she has the right to make the assignment, (ii) the right is not subject to limitations or defenses other than those stated or apparent at the time of assignment, and (iii) she will do nothing to defeat or impair the assigned right. Here, the homeowner did nothing to impair the assignment and the contractor does not appear to have any defenses against the homeowner. Thus, the buyer cannot sue the homeowner. (C) is incorrect because the creditor has no duties under the contract; he just has the right to receive payment. Unless a contrary intention appears, courts will construe language assigning “the contract” as including an assumption of the duties by the assignee. Here, however, the contractor assigned only his right to payment to the creditor and continued to perform the duties under the contract himself. Thus, no delegation of duties will be implied and the creditor cannot be sued for breach.

How well did you know this?
1
Not at all
2
3
4
5
Perfectly
15
Q

A union filed suit against a corporation, known for its antiunion management, asserting that its members were being discharged in retaliation for membership in the union rather than for any failure to perform their jobs properly. Under the pretrial discovery orders, a union employee was allowed to examine all of the records held in the corporation’s files concerning discharge of employees for a seven-year period prior to the instigation of suit by the union. The employee sorted through this large volume of material and discovered that persons who were union activists usually had “lack of corporate spirit” listed as their reason for discharge, while other fired workers tended to have more specific grounds for discharge listed, e.g., persistent lateness. The employee developed a chart showing grounds for dismissal of union members versus nonmembers based on the data in the files. At the trial, the union placed the employee on the stand. She testified in some detail regarding how she had conducted her research. The employee brought out the chart and the union’s lawyer asked that the chart be admitted into evidence. The corporation’s attorney objected.

How should the court rule on the admissibility of the chart?

A. Admissible, because copies of the original documents upon which the chart was based were available to the corporation prior to trial.

B. Admissible, because the chart is helpful to the trier of fact.

C. Inadmissible, because it is hearsay not within any exception.

D. Inadmissible, in the absence of the underlying records having been first introduced into evidence.

A

CORRECT ANSWER: A. Admissible, because copies of the original documents upon which the chart was based were available to the corporation prior to trial.

The chart is admissible because the original documents are in the corporation’s files. The original document or best evidence rule generally requires the original writing to be produced when the terms of the writing are sought to be proved and are material to the case. [Fed. R. Evid. 1002] However, under Federal Rule 1006, the contents of voluminous writings that are otherwise admissible may be presented in the form of a chart as long as the original documents are available to the other party for examination and copying. Here, the underlying documents belonged to the adverse party, and thus the corporation had unlimited access to them. (B) is incorrect because the chart could be helpful to the trier of fact and still be inadmissible, such as if the underlying material were not available to the corporation or the chart were based on inadmissible hearsay. Furthermore, (B) is not as good a choice as (A) because (B) states a generality (it basically states the relevance requirement) whereas (A) applies the law to the specific facts of this case. (C) is incorrect because the chart is admissible provided the underlying documents are admissible. Even if the documents in this case would be hearsay, they would be admissible under the business records exception to the hearsay rule because they are records of events made in the regular course of business. [Fed. R. Evid. 803(6)] (D) is incorrect because Rule 1006 is an exception to the best evidence rule designed to avoid the introduction of voluminous writings into evidence; therefore, it does not require their introduction as a prerequisite to introduction of a chart.

How well did you know this?
1
Not at all
2
3
4
5
Perfectly
16
Q

A city’s water board election laws provide that, although members of the board are elected at large, one member of the board is required to live within each of the five designated water districts within the city. The city’s population was more or less evenly distributed among the districts when this election law was enacted. A resident and registered voter of the city investigated the district residency requirement and discovered that most of the city’s newer residents had moved into the same two water districts, so that the city’s population was no longer evenly distributed among the five water districts. Instead, 80% of the city’s residents lived within its central and eastern water districts, while the other 20% of the city’s residents were scattered among its three other, more rural, districts.

If the resident files suit in federal court challenging the constitutionality of the residency requirement, how will the court most likely rule?

A. The residency requirement is unconstitutional because it impairs the voters’ equal protection rights, in that it gives the voters in the less populous districts more effective representation on the water board.

B. The residency requirement is unconstitutional because it violates the candidates’ equal protection rights.

C. The residency requirement is constitutionally permissible because the water board members do not exercise legislative power.

D. The residency requirement is constitutionally permissible because the water board members are elected at large.

A

CORRECT ANSWER: D. The residency requirement is constitutionally permissible because the water board members are elected at large.

The residency requirement is permissible because the water board is elected at large. The Equal Protection Clause prohibits state dilution of the right to vote, so that when a governmental body establishes voting districts for the election of representatives, the number of persons in each district may not vary significantly. This is known as the principle of “one person, one vote.” This principle applies to almost every election where a person is being elected to perform normal governmental functions (e.g., an election for trustees for a junior college district). However, the principle of one person, one vote generally is inapplicable where there is an at-large system of election (except where the system is adopted for discriminatory purposes). Here, the water board members are elected by all of the qualified voters in the city in an at-large system (rather than having the voters of each individual district select one board member apiece), and no discriminatory intent is evident. Thus, the statutory provision requiring board members to reside in each of the five districts does not result in an imbalance or a dilution of the voting rights of the citizens of the city. Consequently, (A) is incorrect, and (D) presents an accurate statement of the constitutionality of the residency requirement. (Note that the answer might be different under federal statute because the city would have to prove a valid, nondiscriminatory purpose.) (C) is incorrect even though it reaches the correct result. While the Supreme Court has exempted special purpose water storage districts from the one person, one vote requirement, the basis of the decision was the specialized nature of the entity. The constitutional requirements apply not only to legislators, but also to elected administrative and executive officials. (B) is incorrect because, even assuming that the residency requirement violates the candidates’ equal protection rights, the resident would not have standing to raise the issue. Generally, a claimant must assert his own constitutional rights and cannot assert the rights of third parties.

How well did you know this?
1
Not at all
2
3
4
5
Perfectly
17
Q

A landowner entered into a written agreement with a real estate broker whereby the broker would receive a commission of 10% of the sale price if he procured a “ready, willing, and able buyer” for the landowner’s property and if the sale actually proceeded through closing. The broker found a buyer who agreed in writing to buy the property from the landowner for $100,000, the landowner’s asking price. The buyer put up $6,000 as a down payment. The agreement between the landowner and the buyer contained a liquidated damages clause providing that, if the buyer defaulted by failing to tender the balance due of $94,000 at the closing date, damages would be 10% of the purchase price. The landowner included that clause because she was counting on using the proceeds of the sale for a business venture that would likely net her at least $10,000.

The buyer became seriously ill and defaulted. When he recovered, he demanded that the landowner return his $6,000, and the landowner refused. The broker also demanded the $6,000 from the landowner and was refused. The broker and the buyer filed separate suits against the landowner, with the buyer pleading impossibility of performance. The two cases are consolidated into a single case.

How should the court rule as to the disposition of the $6,000?

A. The landowner keeps the entire $6,000, because the liquidated damages clause is reasonable.

B. The buyer gets the entire $6,000, because his performance was impossible.

C. The broker gets the entire $6,000, which is 60% of the commission he is entitled to, because he substantially performed his part of the contract by producing a buyer willing to pay the $100,000 asking price.

D. The broker gets $600 and the landowner gets $5,400, because the damages clause was reasonable and the broker is entitled to 10% of whatever the landowner realizes from the deal.

A

CORRECT ANSWER: A. The landowner keeps the entire $6,000, because the liquidated damages clause is reasonable.

The landowner may keep the $6,000 as liquidated damages. A liquidated damages clause is enforceable if: (i) damages are difficult to ascertain at the time of the making of the contract, and (ii) the damages are a reasonable forecast of compensatory damages. Here, the landowner was unsure what her damages would be if she did not receive the sales proceeds from the property, but $10,000 seemed a reasonable amount. Thus, both criteria for valid liquidated damages clauses are met. (B) is incorrect because impossibility must be objective; i.e., performance cannot be accomplished by anyone. Physical incapacity of a person necessary to effectuate the contract may discharge contractual duties if that person’s performance is clearly impossible. (Usually this occurs in personal services contracts, where only that one person can perform the required duty.) Although the buyer was seriously ill, it is not clear that this made it impossible for him to produce the $94,000. Without more facts, it is reasonable to assume that someone else could have delivered the money or that his mortgage would still have gone through, etc. (C) is incorrect because the conditions for the broker’s payment were not met: It is debatable whether he produced a “ready, willing, and able” buyer, and in any event the sale did not actually proceed through closing. (D) is incorrect because the broker was to receive proceeds from the sale of the property; the $6,000 was damages and not sale proceeds.

How well did you know this?
1
Not at all
2
3
4
5
Perfectly
18
Q

Which of the following suits would not fall within the United States Supreme Court’s original jurisdiction under Article III, Section 2?

A. A suit seeking to assert the interest of state citizens in retaining diplomatic relations with a foreign nation.

B. A suit seeking to protect a state’s timber from allegedly illegal cutting by residents of another state.

C. A suit seeking to enjoin enforcement of an allegedly unconstitutional executive order that will greatly limit the state’s authority to make policy decisions regarding admission to state universities.

D. A suit by the United States Government seeking to enjoin state construction of a bridge over a navigable waterway.

A

CORRECT ANSWER: A. A suit seeking to assert the interest of state citizens in retaining diplomatic relations with a foreign nation.

The suit to assert state citizens’ rights is not within the Supreme Court’s original jurisdiction. Under Article III, Section 2, the United States Supreme Court has original jurisdiction in all cases affecting ambassadors, other public ministers, and consuls, and in which a state is a party. In (A), the state is not really seeking to advance or protect any interest of its own. Rather, the state is attempting to act in parens patriae (i.e., to act as a representative of its citizens, thereby asserting their interests). Thus, the state is not an actual party in this case in the sense that the Supreme Court has traditionally required to justify exercise of original jurisdiction. (B) would be a proper case for institution under the Supreme Court’s original jurisdiction because it involves an attempt by a state to protect its own economic interest rather than to assert the interests of its citizens in a representative capacity. Similarly, (C) sets forth a situation in which a state is attempting to defend its asserted right to render decisions affecting admissions policies relative to its own state universities. Thus, in (C) the state is an actual party to the case. Finally, (D) describes an attempt by the federal government to prevent state construction of a bridge (presumably pursuant to the admiralty power). Clearly, this case involves an alleged grievance that will be directly committed by a state. Therefore, the state is an actual party.

How well did you know this?
1
Not at all
2
3
4
5
Perfectly
19
Q

A man shopping for a leather jacket at a clothing store could not decide between two jackets, so the proprietor, who knew the man and his family well, let him take one of the jackets on approval. No mention was made by the proprietor of the method of payment he expected. The man wore the jacket on a visit to his grandfather, who liked it so much that when the man told him what the jacket cost and that he had taken it on approval, the grandfather said he would buy it for him if he promised to give some of his old clothes to a favorite charity for the poor at Christmastime. The man wholeheartedly agreed to donate the clothes to the charity at Christmas. Very pleased, the grandfather called the shop and told the proprietor to send the bill for the jacket to him, which he did. Before the bill was paid and before the Christmas season arrived, the grandfather fell ill and died. The grandfather’s executor has refused to pay the bill, and the man has not yet given any old clothing to the charity.

Will the proprietor be able to recover the price of the jacket from the estate?

A. Yes, because the proprietor was the intended beneficiary of the promise between the man and his grandfather.

B. Yes, because the man has no duty to give the clothing to the charity.

C. No, because the grandfather’s implied promise to pay the proprietor arising from the phone call is unenforceable.

D. No, because a condition has not yet occurred.

A

CORRECT ANSWER: A. Yes, because the proprietor was the intended beneficiary of the promise between the man and his grandfather.

The proprietor can recover the cost of the jacket from the grandfather’s estate because the proprietor is an intended third-party beneficiary and his right to enforce the contract has vested. The rights of an intended third-party beneficiary vest when the beneficiary (i) manifests assent to the promise in a manner invited or requested by the parties; (ii) brings suit to enforce the promise; or (iii) materially changes his position in justifiable reliance on the promise. Here, the proprietor qualifies as an intended beneficiary of the agreement between the man and his grandfather because the proprietor was expressly designated in the contract, he was to receive performance directly from the grandfather, and he stood in an existing contractual relationship with the man that required the man to either pay for the jacket or return it, making it likely that the young man’s purpose in making the arrangement with his grandfather was to satisfy the obligation to the proprietor. The proprietor can enforce the contract because his rights vested when he sent the bill to the grandfather at the grandfather’s request. Thus, the proprietor will prevail against the grandfather’s estate. (B) is wrong because the man does have a duty to give the clothes to the charity; if he does not do so, he will be in breach of his contract with his grandfather, and this would give his grandfather’s estate a defense to payment. However, the man’s time for performance (Christmastime) has not yet occurred, and so he is not in breach. Nevertheless, this fact is not the reason the proprietor will recover; he will recover due to his status as an intended beneficiary, not because this possible defense has been negated. (C) is wrong because both the result and the rationale are incorrect. The proprietor is not relying on the grandfather’s implied promise to him in the phone call; he is seeking to enforce his rights as a third-party beneficiary of the agreement between the man and his grandfather. Even if the grandfather had not called the proprietor, the proprietor could still have recovered against the grandfather’s estate because of his status as a third-party beneficiary. (D) is wrong because the man’s giving the clothes to the charity is not a condition that must be fulfilled before the grandfather’s estate must pay. The grandfather promised to pay for the jacket if the man promised to donate the clothes; i.e., the consideration for the grandfather’s promise was the man’s promise, not his actually donating the clothes. As soon as the man made the promise, the grandfather’s duty to pay became absolute. (If the man does not donate the clothes, he will be in breach of his contract with his grandfather, but the grandfather’s performance was not conditioned on the man’s donating the clothes.)

How well did you know this?
1
Not at all
2
3
4
5
Perfectly
20
Q

A passenger sued a limo driver for an injury the passenger sustained in an accident in State B, and the passenger had the limo driver promptly served with a summons and complaint. Forty days thereafter, with no answer being filed, the passenger requested and was granted an entry of default by the court clerk, and a date for a hearing for a default judgment was set. Notice of the hearing for the default judgment was sent to the limo driver two weeks prior to the hearing.

Assuming that the hearing for the default judgment has not yet taken place, what is the limo driver’s best method to get a hearing on the merits of the case?

A
Have the entry of default set aside because the clerk did not have the authority to enter a default, since the amount claimed was not for a sum certain.

B
Have the entry of default set aside because no notice was given before the clerk entered the default.

C
Have the entry of the default set aside for good cause.

D
Appear at the hearing for a default judgment.

A

CORRECT ANSWER: A. Have the entry of default set aside because the clerk did not have the authority to enter a default, since the amount claimed was not for a sum certain.

The limo driver’s best approach is to have the entry of default set aside for good cause. Although there are no facts to indicate that good cause exists to have the default set aside, having the default vacated before the hearing on the default judgment represents the only method by which the limo driver can get a hearing on the merits of the case. If the limo driver can have the entry of default vacated prior to the hearing for a default judgment, the hearing would not proceed, and the case would proceed as any other case in which no default was entered. (A) and (B) are incorrect, as both incorrectly state the procedure for obtaining an entry of default. Under Federal Rule of Civil Procedure 55(a), if the defendant does not answer or otherwise defend, the court clerk may enter a default on the record. The default may be entered even if the amount of damages is yet to be determined. No notice to the defendant is required for entry of default. Notice to the defendant of the hearing for a judgment in default is required when the defendant has appeared in some fashion. (D) is incorrect. The entry of the default cuts off the defendant’s right to contest the case on the merits. Although the defendant may appear at the hearing for the default judgment to contest damages, he may not contest liability until the entry of default is vacated.

How well did you know this?
1
Not at all
2
3
4
5
Perfectly
21
Q

A landlord hired a builder to construct a 10-unit apartment building on a single foundation. Within the first year after construction, the floors and walls began to crack. The landlord sued the builder in federal court, alleging that the builder was negligent in constructing the foundation. The builder denied those allegations, and after a year of extensive discovery, the case went to a jury trial, where all issues were litigated.

After closing arguments, the judge presented the jury with a verdict form to complete after deliberations. In addition to instructing the jury to select which party would be granted the verdict, the form also asked several specific questions regarding how the jury decided specific facts in the case.

With what form of verdict was the jury presented?

A
A special verdict form.

B
A general verdict form.

C
A directed verdict form.

D
A general verdict form with interrogatories.

A

CORRECT ANSWER: D. A general verdict form with interrogatories.

The jury was presented with a general verdict form with interrogatories. In such a case, the jury is asked to give a general verdict and also to answer specific questions concerning certain ultimate facts in the case. The purpose is to ensure that the jury properly considered the important issues. Interrogatories must be submitted with the general verdict to test the verdict’s validity. [See Fed. R. Civ. P. 49(b)] Here, the verdict form the judge presented to the jury is a general verdict with interrogatories because it asked the jury to decide ultimately which party prevailed, but also asked specific questions regarding how the jury decided certain facts to test the verdict’s validity. (A) is wrong because it incorrectly classifies the verdict form. In a special verdict, a jury is asked to make a finding on all material conclusions of fact, and the court applies the law. The procedure for a special verdict is to submit to the jury a series of questions regarding each ultimate fact. The court then makes legal conclusions based on those facts. [See Fed. R. Civ. P. 49] Here, although the verdict form did ask specific questions, it also asked the jury to render the verdict, instead of having the court apply the law and reach the legal conclusion. As such, it is not a special verdict form. (B) is wrong because, in a general verdict, the jury finds for the plaintiff or defendant and gives the amount of damages or relief due. Here, the jury was asked to render a verdict for the party, but was also asked specific questions of facts. As such, it was a general verdict with interrogatories. (C) is wrong because a directed verdict is issued by the judge, not the jury, before the case is submitted to the jury. Moreover, this type of verdict is no longer called a “directed verdict.” It is now called a “judgment as a matter of law.” Clearly, this does not apply to the facts, as the case was submitted to the jury.

How well did you know this?
1
Not at all
2
3
4
5
Perfectly
22
Q

A manufacturer sold to a consumer an expensive laser printer that never worked properly. Therefore, the consumer never fully paid for the printer. The manufacturer sued for specific performance of the contract of sale of the printer. The consumer filed a counterclaim for a breach of warranty, asking for $85,000 in damages. The consumer demanded a jury trial, but the manufacturer objected.

Assuming that the demand for a jury trial was timely made, how will the court rule on the availability of a jury trial?

A
For the consumer, because the underlying dispute is legal in nature.

B
For the consumer, because a defendant may always request a jury trial.

C
For the manufacturer, because it filed suit first.

D
For the manufacturer, because an action for specific performance is equitable in nature.

A

CORRECT ANSWER: A. For the consumer, because the underlying dispute is legal in nature.

The court will grant a jury trial because the underlying dispute is legal in nature. The court will look to the basic substance of the case to see if a jury trial is appropriate. Although the manufacturer’s suit is equitable in nature, the consumer’s counterclaim for breach of warranty is an action at law, in which a jury trial is available on demand. Thus, (D) is incorrect. (B) is an incorrect statement of law, and (C) is incorrect because filing suit first would not guarantee a jury (or nonjury) trial.

How well did you know this?
1
Not at all
2
3
4
5
Perfectly
23
Q

The plaintiff and the defendant are both citizens of State A. The plaintiff wished to sue the defendant in a federal court on a $100,000 claim. For this reason, and only this reason, she moved permanently to State B and filed suit in a federal court there, with jurisdiction being based on diversity of citizenship. A few days after being properly served, the defendant, after accepting a job offer from a company based in State B, moved permanently to State B.

For purposes of the plaintiff’s case, what are the citizenships of the parties?

A
Both the plaintiff and the defendant are citizens of State B.

B
The defendant is a citizen of State A, and the plaintiff is a citizen of State B.

C
The defendant is a citizen of State B, and the plaintiff is a citizen of State A.

D
Both the plaintiff and the defendant are citizens of State A.

A

CORRECT ANSWER: B. The defendant is a citizen of State A, and the plaintiff is a citizen of State B.

The defendant is a citizen of State A, and the plaintiff is a citizen of State B. Diversity is determined at the time the action is filed, not when the cause of action accrues or after the action commences. Also, the plaintiff’s motive for moving to State B is irrelevant, as long as the change of citizenship is genuine; i.e., she intends to remain there. The defendant’s motive is also irrelevant. Even though he moved to State B for reasons unrelated to the case and his move appears to be enough to change his citizenship, citizenship is still determined at the time the action is filed, and thus he is still considered to be a citizen of State A for purposes of this lawsuit.

How well did you know this?
1
Not at all
2
3
4
5
Perfectly
24
Q

A woman died, devising a parcel of land by will “to my church, to be held and enjoyed by them so long as they shall maintain and promulgate their present religious beliefs and continue as a church; and if said church shall be dissolved or if its religious beliefs shall be changed, then it is my will that the land shall go to my children, to be divided in equal portions among them.” The residuary clause of the woman’s will provides “the residue of my estate I bequeath to my children.” The common law Rule Against Perpetuities applies in this jurisdiction.

If the church dissolves, by which manner will the children take the land?

A
Under the residuary clause of the woman’s will by making an entry upon the land.

B
Under the residuary clause of the woman’s will, because a possibility of reverter is devisable.

C
By intestacy from the woman’s estate, because their executory interest violates the Rule Against Perpetuities.

D
Pursuant to their executory interest in the land.

A

CORRECT ANSWER: Under the residuary clause of the woman’s will, because a possibility of reverter is devisable.

Answer B is correct. The children will take the land by using the possibility of reverter that passed to them under the residuary clause of the woman’s will. In this case, the woman conveyed a fee simple subject to an executory interest to the church. While she attempted to make a gift over to her children, this was an invalid executory interest because it was unlimited in time and therefore violated the Rule Against Perpetuities. When the executory interest is stricken, the grant becomes a fee simple determinable, and the woman’s estate retains possibility of reverter. A possibility of reverter, is a future interest in a grantor and thus is not subject to the Rule Against Perpetuities. That interest is devisable and passed through the residuary clause of her will to her children. That possibility of reverter became possessory when the church dissolved. Answer A is incorrect. Once the executory interest was stricken, the church held a fee simple determinable, not a fee simple subject to a condition subsequent. The latter leaves a right of entry in the grantor, where the former leaves a possibility of reverter in the grantor. Therefore the children need not make an entry onto the land in order to take. Upon the happening of the condition, the estate is terminated and possibility of reverter immediately reverts the estate to the grantor—there is no need for making entry. Answer C is incorrect. As explained above, a possibility of reverter is not subject to the Rule Against Perpetuities. Also, that interest is devisable and passed through the residuary clause of her will to her children. That possibility of reverter became possessory when the church dissolved. Answer D is incorrect. The children’s executory interest was stricken. An executory interest is subject to the Rule Against Perpetuities, and this interest violated the Rule. This executory interest was unlimited in the time when it could become possessory and therefore vest. The church could clearly operate long beyond lives in being plus 21 years.

How well did you know this?
1
Not at all
2
3
4
5
Perfectly
25
Q

A farmer conveyed his farm to his son for life, then to the heirs of his daughter (a living person).

How can the interest in the heirs of the daughter be characterized?

A
Springing executory interest, if the daughter dies before the son.

B
Contingent remainder before the death of the daughter, because those who will take have not been ascertained.

C
Vested remainder, if the son dies before the daughter.

D
Contingent remainder before the death of the son, because the heirs of the daughter must survive the son in order to take.

A

CORRECT ANSWER: B. Contingent remainder before the death of the daughter, because those who will take have not been ascertained.

Answer B is correct. The daughter’s heirs have a contingent remainder before the daughter’s death because they have not yet been ascertained. The interest in the daughter’s heirs is a remainder because it naturally follows the life estate in the son, but it is contingent until the daughter’s death, because until her death, the heirs cannot be determined. The daughter’s heirs must survive her in order to be considered heirs. Thus, survival past the daughter’s death is a condition precedent to the vesting of this interest. Answer A is incorrect. A springing executory interest occurs when there is a gap between the time when a present possessory interest ends and the future interest becomes possessory. For example, in this case, if the son died before the daughter, there would be no heirs of the daughter at that time, because heirs are not ascertained until death. Therefore, the interest would revert to the grantor until the daughter died and her heirs were ascertained, at which time the interest would “spring” from the grantor to the heirs. This answer choice is incorrect, because there is no gap if the daughter dies before the son, for the daughter’s heirs would be ascertained at her death and would be able to take possession of the land at the son’s death. Answer C is incorrect. The interest in the “heirs” of the daughter will not vest until her death, because they are unascertained until that time. If the son predeceases the daughter, the contingent remainder held by the descendants of the daughter will be transformed into a springing executory interest which will become possessory at the death of the daughter, which is when the identity of her heirs will be determined. Therefore, the interest in the daughter’s heirs does not vest at the son’s death, because the interest is not contingent on those persons surviving the son. Rather, the interest is contingent until those persons survive the daughter. Answer D is incorrect. Although Answer D correctly states that the daughter’s heirs hold a contingent remainder, the answer choice does not correctly state why the interest is contingent. The interest is contingent because the daughter’s heirs cannot be ascertained until she dies. They need not survive the son in order to take an interest; they need only survive the daughter. At that point, their interests vest. If an heir of the daughter survives her, but predeceases the son, the heir’s interest will pass to the heir’s estate.

How well did you know this?
1
Not at all
2
3
4
5
Perfectly
26
Q

A man and a woman are tenants in common in a condominium. The man spent $50,000 on improvements to the condominium, which increased its market value by $30,000.

If the man sues the woman for reimbursement, what amount will he receive?

A
$30,000.

B
$25,000.

C
$15,000.

D
Nothing.

A

CORRECT ANSWER: D. Nothing

Answer D is correct. The man will receive nothing. A co-tenant who makes necessary repairs to property can usually recover from a noncontributing co-tenant, but a co-tenant who improves property has no right to contribution from noncontributing co-tenants. At best, he can recover the value of those improvements in an action for partition.

How well did you know this?
1
Not at all
2
3
4
5
Perfectly
27
Q

A landlord leased an apartment to a salesman for two years at a rent of $1200 a month. Rental payment was due on the first day of each month. The terms of the lease prohibited any assignment without written consent of the landlord.

The salesman paid rent for one year but was then transferred to another state by his employer for six months. The salesman entered into an agreement with a cousin for the cousin to move into the apartment and pay the full amount of the rent for the six months the salesman would be gone. Because the salesman was worried that he’d be too busy with his new job to remember to make the rent payments, he asked the cousin to pay the rent directly to the landlord. The cousin paid the rent for the first four months, and the landlord accepted the rent. However, the cousin did not pay any rent for the final two months.

When the salesman returned and took possession, he promptly made a payment of $1200 to the landlord. The landlord notified him that he was bringing suit for the $2400 of unpaid rent for the previous two months.

From whom can the landlord recover this amount?

A
From the cousin only, based on privity of estate.

B
From the cousin only, because the landlord consented to the sublease by accepting the four months’ rent.

C
From the salesman only, based on privity of contract and privity of estate.

D
From either the salesman or the cousin, because the lease agreement did not expressly prohibit a sublease.

A

CORRECT ANSWER: C. From the salesman only, based on privity of contract and privity of estate.

(C) is the correct answer. The landlord can recover from the salesman only. The transfer from the salesman to the cousin was a sublease, not an assignment, because it was for less than the entire remaining leasehold balance. In a sublease, the original tenant remains liable both in privity of contract and in privity of estate. The landlord may recover unpaid rent only from the original tenant. (A) is incorrect because there is no privity of estate between a landlord and sublessee because the sublessee does not hold the tenant’s full estate. (B) is incorrect because even if it is true, it would not allow the landlord to recover from the cousin. The problem is not the landlord’s consent; it is privity. Assuming that the landlord’s acceptance of the rent from the cousin was a waiver of a covenant against sublease, the landlord has still no privity of estate or contract with the sublessee, and thus cannot collect rent from the cousin. (D) is incorrect because, even if true, this fact would not affect the landlord’s ability to sue the cousin. If the salesman was not permitted to sublease and did so in violation of his lease with the landlord, it would not give the landlord rights against the cousin.

How well did you know this?
1
Not at all
2
3
4
5
Perfectly
28
Q

A landlord entered into a written residential lease with a tenant on June 1. The premises consist of a single-family house and surrounding land. The tenant paid the first and last months’ rent at the time the lease was signed. The lease term was for one year, commencing July 1. At the time the lease was signed, the premises were occupied by a writer under a lease expiring on June 30. The writer remained in possession after his lease expired, leaving only after being evicted on August 15.

What is the tenant’s best theory for a cause of action against the landlord for failure to deliver possession on or before July 1?

A
The landlord breached the covenant of quiet enjoyment, which is implied in every lease.

B
The writer’s continued possession constituted a constructive eviction of the tenant.

C
The landlord had an immediate right to possession on July 1 and should have physically removed the writer on that date.

D
The landlord negligently failed to remove the writer on or before July 1.

A

CORRECT ANSWER: A. The landlord breached the covenant of quiet enjoyment, which is implied in every lease.

Answer A is correct. The tenant’s best theory is that the landlord breached the covenant of quiet enjoyment. A covenant of quiet enjoyment is implied in every lease. The tenant paid rent for a term commencing July 1 and had a right to possession on that date. The landlord breached that covenant by permitting a person with a greater possessory right (e.g., a hold-over tenant at sufferance who has not yet been evicted) to deny possession to the tenant, who had a right to it. Answer B is incorrect. Constructive eviction occurs when a landlord’s breach of duty renders the premises untenantable (i.e., unsuitable for occupancy). This usually occurs when the landlord fails to repair or provide necessary services, such as heat or water. This tenant has never been in possession of the premises and thus cannot be evicted, constructively or otherwise. Answer C is incorrect. This answer choice correctly states that the landlord had a right to possession on July 1. However, it incorrectly states that the landlord should have resorted to self-help to regain possession. The writer was originally a tenant under a lease and became a tenant at sufferance at the expiration of the lease. His possession could be terminated only by a court-ordered eviction, not by self-help repossession. The tenant’s best theory is that he has a valid lease with the landlord that contains a covenant of quiet enjoyment, and that covenant was breached by the fact that someone claiming through the landlord prevented the tenant from taking possession of the premises at the start of the lease. Answer D is incorrect. A claim of negligence would not be the best argument for the tenant, because it appears that the landlord acted diligently and quickly to pursue the legal means available to remove a hold-over tenant at sufferance. The tenant should instead pursue a theory of breach of the implied covenant of quiet enjoyment, since he was denied his legal right to possession by the continued possession of the writer.

How well did you know this?
1
Not at all
2
3
4
5
Perfectly
29
Q

A landlord leased a 40-acre tract of land to a tenant for a 15-year period. After five years had expired, the government condemned 15 acres of the property for road construction and allocated the compensation award to the landlord and the tenant according to their respective interests. It so happened, however, that the tenant had used the 15 acres taken by the government to store vehicles necessary in the tenant’s work. The tenant knew of no other place nearby where he could store the vehicles. There is no applicable statute in the jurisdiction where the property is located, nor any provision in the lease relating to the condemnation. The tenant quit possession, claiming that he could no longer live on the premises if he could not park the vehicles needed in his work close to where he lived. The landlord brought suit against the tenant to recover rent.

Is the landlord likely to prevail?

A
Yes, because the relationship of landlord and tenant was unaffected by the condemnation, thus leaving the tenant still obligated to pay rent.

B
Yes, because of the implied warranty on the part of the tenant to return the demised premises in the same condition at the end of the term as it was at the beginning.

C
No, because there has been a breach of the implied covenant of quiet enjoyment by the landlord’s inability to provide the tenant with possession of the whole of the property for the entire term.

D
No, because there has been a frustration of purpose that excuses the tenant from further performance of his contract to pay rent.

A

CORRECT ANSWER: A. Yes, because the relationship of landlord and tenant was unaffected by the condemnation, thus leaving the tenant still obligated to pay rent.

The landlord likely will prevail. In a partial condemnation case, the landlord-tenant relationship continues, as does the tenant’s obligation to pay the entire rent for the remaining lease term. (B) is wrong because, while the tenant generally is obligated to return the premises in the same condition as when received, that obligation would not be considered breached by the actions of a third party such as the government. (C) is wrong because the covenant of quiet enjoyment can be breached only by actions of the landlord and not those of a third party, such as the government. (D) is wrong because the law of landlord and tenant traditionally refuses to recognize frustration of purpose as grounds for termination of a lease.

How well did you know this?
1
Not at all
2
3
4
5
Perfectly
30
Q

A landlord leased a portion of the lobby of his office building to a vendor under a written lease for a term of 10 years at $500 per month. The lease provided that the shop was to be used as a candy and cigarette counter, which would be the “exclusive” vending facility in the building. The lease was silent as to the vendor’s right to transfer the leasehold interest.

Later, the vendor transferred his interest to a retailer by a writing providing that the vendor could reenter if the retailer failed to make rental payments to the landlord. The landlord subsequently sold the building to an investor, who installed several automatic candy and cigarette vending machines throughout the building. As a result, the retailer’s sales declined and she refused to pay any rent.

If the investor files suit against the retailer for the rent due, how should a court following common law principles rule?

A
The investor has no direct claim against the retailer personally; his only remedy is to terminate the lease and sue the vendor for the rent.

B
The investor can recover $500 per month rent from the retailer.

C
The investor can recover nothing from the retailer because his installation of the vending machines allowed the retailer to immediately terminate her duties under the lease.

D
The investor can recover $500 per month rent from the retailer but only if the vending machines are removed.

A

CORRECT ANSWER: A. The investor has no direct claim against the retailer personally; his only remedy is to terminate the lease and sue the vendor for the rent.

The investor can recover $500 per month rent from the retailer. The retailer is liable to the investor for the rent because of her status as an assignee, and because at common law the rent and anticompetition covenants are independent. Absent an express restriction in the lease, a tenant may freely transfer his leasehold interest. To be an assignment, a transfer must be on the same terms as the original leasehold except that the transferring tenant may reserve a right of termination (reentry) for breach of the terms of the original lease that has been assigned. Because the vendor transferred all of his interest to the retailer, this transfer will be given effect as an assignment rather than a sublease despite the vendor’s reservation of a right of reentry. An assignee takes the place of the original tenant in a direct relationship with the landlord. The assignee and landlord are in privity of estate, so that each is liable to the other on all lease covenants that run with the land. Covenants held to run with the land include covenants to pay money. Because a covenant to pay rent runs with the land, an assignee owes the rent directly to the landlord for the time that they are in privity of estate. The investor and the retailer are in privity of estate. Thus, the burden of the vendor’s original covenant to pay rent runs with the land and binds the retailer. (A) incorrectly states that the investor has no direct claim against the retailer personally for payment of the rent. (C) and (D) are wrong because, at common law, a tenant’s duty to pay rent is considered an obligation independent of the landlord’s performance of his obligations. The landlord’s original agreement that the space leased to the vendor would be the exclusive vending facility in the building ran with the land. The provision would certainly have been intended to run with the land (it is highly doubtful that the vendor would have intended that a successor in interest to the landlord could lease space for competing vending facilities). Also, this agreement touched and concerned the leased premises because it benefited the tenant and burdened the landlord with respect to their interests in the property. The investor has breached this covenant by installing the vending machines. This breach by the investor will not, however, allow the retailer to terminate the lease or her obligation to pay rent under the lease; i.e., the retailer’s duty to pay rent is independent of the investor’s performance of his agreement that there would be no other vending facilities in the building. Thus, the investor can recover the rent payments regardless of whether the vending machines are removed, and the retailer’s remedy for the investor’s breach is to bring an action against the investor for any damages resulting from his installation of the vending machines.

How well did you know this?
1
Not at all
2
3
4
5
Perfectly
31
Q

If a mortgagee purports to transfer the mortgage without the note, which of the following is not a possible result?

A
The transfer would be void

B
The note would automatically follow the mortgage

C
Payment of the mortgage debt would become due in full

A

CORRECT ANSWER: C. Payment of the mortgage debt would become due in full.

If a mortgagee purports to transfer the mortgage without the note, payment of the mortgage debt would NOT become due in full. Rather, that would be the effect if the mortgagor transferred land subject to a mortgage containing a due-on-sale clause. Such clauses benefit the mortgagee, protecting it from the mortgagor’s sale to a poor credit risk or to a person likely to commit waste.

Mortgagees usually transfer their interests by indorsing and delivering the promissory note and executing a separate assignment of the mortgage. The note and mortgage must pass to the same person for a mortgagee’s transfer to be complete. If a mortgagee purports to transfer the mortgage without the note, some states hold that the note would automatically follow the mortgage. Other states hold that, because the note is the principal evidence of the debt, the transfer would be void.

How well did you know this?
1
Not at all
2
3
4
5
Perfectly
32
Q

When a mortgagee transfers a promissory note, which of the following is required for the transferee to become a holder in due course?

A
The payee must indorse the original note or a copy thereof

B
The note must not contain an acceleration clause

C
The note must provide for a fixed interest rate

D
The transferee must not have any notice that the note is overdue

A

CORRECT ANSWER: D. The transferee must not have any notice that the note is overdue

When a mortgagee transfers a promissory note, to become a holder in due course the transferee must not have any notice that the note is overdue or has been dishonored. The primary benefit of holder in due course status is that a holder in due course takes the note free of any personal defenses that the maker might raise and is subject only to “real” defenses, such as infancy, duress, and illegality.

When a mortgagee transfers a promissory note, for the transferee to become a holder in due course:

  1. The note must be negotiable in form—i.e., the note must be payable to bearer or to the order of the named payee and must contain a promise to pay a fixed amount of money (although it MAY provide for an adjustable interest rate) and no other promises (although it MAY contain an acceleration clause and an attorneys’ fee clause); 2. The named payee must indorse (i.e., sign) the original note—NOT a copy thereof; 3. The original note must be delivered to the transferee; and 4. The transferee must pay value for the note and take the note in good faith, without notice that the note is overdue or has been dishonored, or that the maker has any defense to the duty to pay it.
How well did you know this?
1
Not at all
2
3
4
5
Perfectly
33
Q

When a mortgagee transfers a promissory note, which of the following is required for the transferee to become a holder in due course?

A
The transferee may have notice of defenses only if the note has not been dishonored

B
The transferee must pay fair market value for the note

C
A copy of the note must be delivered to the transferee

D
The note must be payable to bearer or to the order of the named payee

A

CORRECT ANSWER: D. The note must be payable to bearer or to the order of the named payee

When a mortgagee transfers a promissory note, for the transferee to become a holder in due course the note must be payable to bearer or to the order of the named payee. The primary benefit of holder in due course status is that a holder in due course takes the note free of any personal defenses that the maker might raise and is subject only to “real” defenses, such as infancy, duress, and illegality.

When a mortgagee transfers a promissory note, for the transferee to become a holder in due course:

  1. The note must be negotiable in form—i.e., the note must be payable to bearer or to the order of the named payee and must contain a promise to pay a fixed amount of money (although it may provide for an adjustable interest rate) and no other promises (although it may contain an acceleration clause and an attorneys’ fee clause); 2. The named payee must indorse (i.e., sign) the original note; 3. The original note must be delivered to the transferee (a copy is not acceptable); and 4. The transferee must pay “value” for the note (which must be more than nominal but need not be as great as the fair market value of the note) and take the note in good faith, with NO notice that the maker has any defense to the duty to pay it or that the note is overdue or has been dishonored.
How well did you know this?
1
Not at all
2
3
4
5
Perfectly
34
Q

Under __________, a mortgagee may not take possession of the mortgaged property without consent upon the mortgagor’s default.

A
The intermediate theory

B
Both the lien theory and the title theory

C
The title theory

D
The lien theory

A

CORRECT ANSWER: D. The lien theory

Under the lien theory, a mortgagee may not take possession of the mortgaged property without consent upon the debtor’s default. A majority of the states follow the lien theory, under which the mortgagee is deemed to hold a security interest in the land and the mortgagor is considered the owner until foreclosure. Thus, the mortgagee may not take possession of the land before foreclosure unless the mortgagor consents or has abandoned the land.

Under the title theory, followed in a minority of the states, legal title is in the mortgagee until the mortgage has been satisfied or foreclosed. Thus, the mortgagee is entitled to possession upon demand at any time, which means the mortgagee can take possession as soon as the mortgagor defaults. The same is true in the few states that follow the intermediate theory; however, under this theory, legal title transfers from the mortgagor to the mortgagee on default

How well did you know this?
1
Not at all
2
3
4
5
Perfectly
35
Q

For which type of security interest in land does the debtor transfer title to land to a lender in exchange for a lease with an option to repurchase?

A
Right of first refusal

B
Installment land contract

C
Sale-leaseback

D
Deed of trust

A

CORRECT ANSWER: C. Sale-leaseback

A sale-leaseback is a security interest in land by which the debtor transfers title to land to a lender in exchange for a lease with an option to repurchase. Additional factors that will lead a court to treat the transaction as a disguised mortgage are: (i) similarity between the lease payments and payments that would be due on a mortgage loan; and (ii) the repurchase option being far below the property value, thus making repurchase very likely to occur.

A deed of trust is a security interest in land by which the debtor (i.e., the trustor) transfers title to the land to a third party (i.e., the trustee), such as the lender’s lawyer or a title insurance company, acting on behalf of the lender (i.e., the beneficiary). In the event of default, the lender instructs the trustee to foreclose the deed of trust by selling the property.

An installment land contract is a security interest in land in which the debtor (i.e., the buyer) contracts with the seller to pay for the land in regular installments until the full contract price has been paid, plus interest. Only then will the seller transfer legal title to the buyer. The contract may contain a forfeiture clause providing that the seller may cancel the contract upon default, retain all money paid, and retake possession of the land.

A right of first refusal is not a security interest in land. It is the holder’s preemptive right to purchase property only if the seller accepts a third party’s offer to purchase, and usually on the same terms as that offer.

How well did you know this?
1
Not at all
2
3
4
5
Perfectly
36
Q

A purchase money mortgage (“PMM”), whether recorded or not, generally will have priority over __________.

A
all other mortgages and liens

B
a subsequent recorded non-PMM against the same mortgagor

C
a prior claim against the same mortgagor arising before the mortgagor acquired title

D
no other mortgages or liens

A

CORRECT ANSWER: C. a prior claim against the same mortgagor arising before the mortgagor acquired title

A purchase money mortgage (“PMM”), whether recorded or not, generally will have priority over a prior claim against the same mortgagor arising before the mortgagor acquired title. A PMM is a mortgage given to (i) the vendor of the property as a part of the purchase price, or (ii) a third party who lends funds to allow the buyer to purchase the property. A PMM has priority over such prior claims even if those other mortgages or liens are recorded first.

A PMM that was not recorded generally will not have priority over a subsequent recorded mortgage or lien, such as a non-PMM, against the same mortgagor. PMM priority may be defeated by subsequent mortgages or liens by normal operation of the recording acts. In general, if a first mortgagee fails to record and the second mortgagee records, gives value, and takes without notice of the first, any recording act will give the second mortgagee priority over the first. Thus, a PMM does not have priority over all other mortgages and liens. Furthermore, as between a vendor PMM and a third-party PMM, the vendor’s usually is given priority.

How well did you know this?
1
Not at all
2
3
4
5
Perfectly
37
Q

If a grantor transfers a deed in exchange for cash, and the grantee promises to return the land when the cash is repaid, a court will likely treat the transaction as:

A
An installment land contract

B
A deed of trust

C
An equitable mortgage

D
A conveyance

A

CORRECT ANSWER: C. An equitable mortgage.

If a grantor transfers a deed in exchange for cash, and the grantee promises to return the land when the cash is repaid, a court will likely treat the transaction as an equitable mortgage and not as a conveyance of the land outright. In determining whether the parties intended the transfer only to serve as security for an obligation, the court will consider:

(i) The existence of a debt or promise of payment by the grantor; (ii) The grantee’s promise to return the land if the debt is paid; (iii) Whether the amount advanced to the grantor was much lower than the value of the property; (iv) The degree of the grantor’s financial distress; and (v) The parties’ prior negotiations.

If the court concludes that the deed was given as security, the grantee-creditor must foreclose it like any other mortgage.

A deed of trust is a security interest in land by which the debtor (i.e., the trustor) transfers title to the land to a third party (e.g., the lender’s lawyer or a title insurance company) as trustee for the lender ( i.e., the beneficiary). In the event of default, the lender instructs the trustee to foreclose the deed of trust by selling the property.

An installment land contract is a security interest in land by which the debtor agrees to make regular installment payments until the full contract price (including interest) has been paid. Only then will the vendor transfer legal title to the debtor. In the event of default, the contract may contain a forfeiture clause providing that the vendor may cancel the contract, retain all money paid to date, and retake possession of the land.

How well did you know this?
1
Not at all
2
3
4
5
Perfectly
38
Q

Which correctly states the order of priority for allocating mortgage foreclosure sale proceeds, from first to last?

A
The foreclosing party, any junior lienors in the order of their priority, and then the mortgagor

B
The foreclosing party, any senior lienors in the order of their priority, and then the mortgagor

C
The foreclosing party, any senior lienors in the order of their priority, and then any junior lienors in the order of their priority

D
Any senior lienors in the order of their priority, the foreclosing party, and then any junior lienors in the order of their priority

A

CORRECT ANSWER: A. The foreclosing party, any junior lienors in the order of their priority, and then the mortgagor

The order of priority for allocating mortgage foreclosure sale proceeds is as follows, from first to last: 1. Expenses of the sale, including attorneys’ fees, and court costs; 2. The principal and accrued interest on the foreclosing party’s loan; 3. Any junior lienors in the order of their priority; and then 4. The mortgagor. In many cases, no surplus remains after the principal debt is paid off. Senior lienors receive none of the proceeds. Because a senior lien remains on the property (i.e., may itself be foreclosed in the future), a senior lienor is not entitled to any of the money from the sale, even if there is a surplus.

How well did you know this?
1
Not at all
2
3
4
5
Perfectly
39
Q

If a debtor deeds land to her creditor in exchange for cash, may a court treat the conveyance as a security interest?

A
No, unless the parties executed a mortgage contract

B
Yes, if the creditor agreed to accept the land as payment in full for the loan

C
Yes, if the creditor promised to return the land when the cash is repaid in full

D
No, regardless of the parties’ prior negotiations

A

CORRECT ANSWER: C. Yes, if the creditor promised to return the land when the cash is repaid in full

Yes, if a debtor deeds land to her creditor in exchange for cash, a court may treat the conveyance as a security interest if the creditor promised to return the land when the cash is repaid in full. A court will likely treat such a transaction as an equitable mortgage and not as a conveyance of the land outright. In determining whether the parties intended the transfer only to serve as security for an obligation, the court will consider:

(i) The existence of a debt or promise of payment by the grantor; (ii) The grantee’s promise to return the land if the debt is paid; (iii) Whether the amount advanced to the grantor was much lower than the value of the property; (iv) The degree of the grantor’s financial distress; and (v) The parties’ prior negotiations.

Considering the above, a court is not likely to treat the conveyance as a security interest if the creditor agreed to accept the land as payment in full for the loan.

If the court thus concludes that the deed was given as security, the grantee-creditor must foreclose it like any other mortgage. It will be treated as an equitable mortgage even if the parties did NOT execute a mortgage contract.

40
Q

May the vendor of an installment land contract containing a forfeiture clause reinstate strict performance once she has waived it?

A
Yes, by giving the purchaser notice and a reasonable time to catch up on payments

B
No; a vendor’s waiver is permanent

C
Yes, by establishing a pattern of accepting on-time payments

A

CORRECT ANSWER: A. Yes, by giving the purchaser notice and a reasonable time to catch up on payments

Yes, the vendor of an installment land contract containing a forfeiture clause may reinstate strict performance once she has waived by giving the purchaser notice and a reasonable time to catch up on payments. In an installment land contract, the debtor (i.e., the purchaser) contracts with the vendor to pay for the land in regular installments until the full contract price has been paid, plus interest. Only then will the vendor transfer legal title to the purchaser. The contract may contain a forfeiture clause providing that the vendor may cancel the contract upon default, retain all money paid, and retake possession of the land (instead of foreclosing). Because forfeiture is such a harsh remedy, courts have tended to resist enforcing forfeiture clauses by, for example, finding a waiver of strict performance when a vendor has established a pattern of accepting late payments from the purchaser. To reinstate strict performance, the vendor must notify the purchaser of her intent to do so and must allow the purchaser a reasonable time to make up any late payments.

Although establishing a pattern of accepting late payments constitutes waiver of strict performance, establishing a pattern of accepting on-time payments does not automatically reinstate strict performance. However, a vendor’s waiver is NOT permanent; strict performance may be reinstated as explained above.

41
Q

Mortgages senior to the one being foreclosed are __________ by the foreclosure, and thus a senior mortgagee __________.

A
not affected; maintains its lien on the property after the sale

B
redeemed; may pay off the junior mortgage after default

C
satisfied; is entitled to the proceeds of sale

D
extinguished; is a necessary party to the foreclosure action

A

CORRECT ANSWER: A. not affected; maintains its lien on the property after the sale

Mortgages senior to the one being foreclosed are not affected by the foreclosure, and thus a senior mortgagee maintains its lien on the property after the sale. Foreclosure destroys all interests (e.g., liens, mortgages, leases, easements) junior to the mortgage being foreclosed. Thus, if a lien senior to a mortgagee’s is in default, that mortgagee has the right to pay it off (i.e., redeem it) to avoid being wiped out by its foreclosure. Because interests junior to the mortgage being foreclosed are extinguished by foreclosure, a junior mortgagee is a necessary party to the foreclosure action. Failure to include a necessary party results in the preservation of that party’s interest despite foreclosure and sale.

Mortgages senior to the one being foreclosed are not satisfied by the foreclosure; as is explained above, the senior mortgage remains on the property after the sale. Because its lien remains on the land (i.e., may itself be foreclosed in the future), the senior mortgagee is not entitled to the proceeds of sale, even if there is a surplus.

Mortgages are not redeemed by foreclosure. Redemption is the act of paying off the mortgage debt, thus freeing the land of the mortgage and preventing a foreclosure action. A mortgagor may redeem the land in equity prior to the foreclosure sale. While a senior mortgagee may pay off the junior mortgage after default, there is no reason for it to do so. The property will remain subject to the senior mortgage even if the junior mortgagee forecloses and the land is purchased by a new buyer at the foreclosure sale. By contrast, a junior mortgagee has an incentive to pay off a defaulting senior mortgage, as the foreclosure of that mortgage would wipe out the junior interest in the land.

42
Q

When allocating proceeds from a mortgage foreclosure sale, which of the following receives payment last?

A
Senior liens

B
Junior liens

C
Expenses of the sale

D
The mortgagor

A

CORRECT ANSWER: D. The mortgagor.

When allocating proceeds from a mortgage foreclosure sale, the mortgagor receives payment last. The priority of the proceeds from a foreclosure sale is as follows:

  1. Expenses of the sale, attorneys’ fees, and court costs; 2. The principal and accrued interest on the loan that was foreclosed; 3. Any junior liens or other junior interests in the order of their priority; and then 4. The mortgagor.

In many cases, no surplus remains after the principal debt is paid off.

When allocating proceeds from a mortgage foreclosure sale, senior liens receive none of the proceeds. Because a senior lien remains on the land ( i.e., may itself be foreclosed in the future), a senior lienor is not entitled to any of the money from the sale, even if there is a surplus.

43
Q

Are junior lienors entitled to any proceeds of a mortgage foreclosure sale?

A
No, because junior interests remain on the property after the sale

B
No, because junior interests are extinguished by foreclosure

C
Yes, but only after expenses of the sale, attorneys’ fees, court costs, and the foreclosing party’s loan have been paid

D
Yes, but only after expenses of the sale, attorneys’ fees, court costs, and the mortgagor have been paid

A

CORRECT ANSWER: C. Yes, but only after expenses of the sale, attorneys’ fees, court costs, and the foreclosing party’s loan have been paid

Yes, junior lienors are entitled to proceeds of a mortgage foreclosure sale, but only after expenses of the sale, attorneys’ fees, court costs, and the foreclosing party’s loan have been paid. The priority of the proceeds from a foreclosure sale is as follows:

  1. Expenses of the sale, attorneys’ fees, and court costs; 2. The principal and accrued interest on the foreclosing party’s loan; 3. Any junior lienors or other junior interests in the order of their priority; and then 4. The mortgagor.

Although junior interests are extinguished by foreclosure, junior lienors are entitled to any surplus—i.e., proceeds remaining after the principal debt is paid. In many cases, however, no surplus remains after the principal debt is paid.

Junior interests do NOT remain on the property after the sale; they are extinguished by foreclosure if they are properly made parties to the foreclosure proceeding. In contrast, foreclosure does not affect any interest senior to the mortgage being foreclosed, which remains on the property.

44
Q

__________ is the right of a mortgagor to recover the land by paying the foreclosure sale price after the foreclosure sale has occurred.

A
Power of sale

B
Equitable redemption

C
Statutory redemption

A

CORRECT ANSWER: C. Statutory redemption.

Statutory redemption is the right of a mortgagor to recover the land by paying the foreclosure sale price after the foreclosure sale has occurred. About half the states provide a statutory right to redeem for some fixed period after the foreclosure sale has occurred, usually six months or one year. The amount to be paid is the foreclosure sale price, rather than the amount of the original debt. This right extends to mortgagors and, in some states, to junior lienors.

Equitable redemption, in contrast, is the right of a mortgagor to recover the land by paying the amount overdue, plus interest, at any time before the foreclosure sale. If the mortgagor has defaulted and the mortgage or note contained an acceleration clause, then the full balance must be paid in order to redeem in equity.

A power of sale does not refer to the mortgagor’s right to redeem the land. Rather, a “power of sale” clause authorizes the trustee in a deed of trust to advertise, give appropriate notices, and conduct a nonjudicial foreclosure sale personally. All states allow mortgages to be foreclosed by judicial sale, while about one-half also allow nonjudicial sale under a power of sale for deeds of trust.

45
Q

When allocating proceeds from a mortgage foreclosure sale, which of the following has priority?

A
The mortgagor

B
Junior lienors

C
Court costs

D
The foreclosing party

A

CORRECT ANSWER: C. Court costs.

When allocating proceeds from a mortgage foreclosure sale, court costs have priority. The priority of the proceeds from a foreclosure sale is as follows:

  1. Expenses of the sale, attorneys’ fees, and court costs; 2. The principal and accrued interest on the foreclosing party’s loan; 3. Any junior lienors or other junior interests in the order of their priority; and then 4. The mortgagor.

In many cases, no surplus remains after the principal debt is paid off.

46
Q

When may a mortgagee take possession of the mortgaged property?

A
On default, in a title theory state.

B
Never under any circumstances.

C
As soon as the mortgage is created.

D
On default, in a lien theory state.

A

CORRECT ANSWER: A. On default, in a title theory state.

A mortgagee may take possession of the mortgaged property on default, in a title theory state. When a mortgagor defaults on his debt, the mortgagee can sue on the debt or foreclose the mortgage. The mortgagee will have a right to take possession before foreclosure only in the minority of jurisdictions where she is deemed to have legal title to the property. In practice, this means the mortgagee can take possession as soon as a default occurs.

A mortgagee may not take possession of the mortgaged property on default in a lien theory state. A majority of the states follow the lien theory, under which the mortgagee is deemed to hold a security interest in the land and the mortgagor is considered the owner until foreclosure. According to this theory, the mortgagee may not take possession of the land before foreclosure.

A mortgagee may not take possession of the mortgaged property as soon as the mortgage is created. In both title theory and lien theory states, discussed above, the mortgagee cannot take possession until the mortgagor defaults. Thus, if the mortgagor duly pays the debt, the mortgagee will not be entitled to possession. The same is true in the few jurisdictions that follow the intermediate theory, in which legal title transfers from the mortgagor to the mortgagee on default.

It is not correct that a mortgagee may never under any circumstances take possession of the mortgaged property. In all states, a mortgagee may take possession if the mortgagor consents or abandons the land.

47
Q

May a vendor who has accepted late payments for six months on an installment land contract declare a forfeiture if payment is late the seventh month?

A
Yes, and the purchaser also is liable for damages

B
Yes, if the contract provides for forfeiture in the event of default

C
No, because the vendor has waived strict performance of the contract

D
No, because forfeiture clauses are void in all jurisdictions

A

CORRECT ANSWER: C. No, because the vendor has waived strict performance of the contract

No, a vendor who has accepted late payments for six months on an installment land contract may not declare a forfeiture if payment is late the seventh month, because the vendor has waived strict performance of the contract. In an installment land contract, the debtor (i.e., the purchaser) contracts with the vendor to pay for the land in regular installments until the full contract price has been paid, plus interest. Only then will the vendor transfer legal title to the purchaser. The contract may contain a forfeiture clause providing that the vendor may cancel the contract upon default, retain all money paid, and retake possession of the land (instead of foreclosing). Forfeiture clauses are NOT void in all jurisdictions, but because forfeiture is such a harsh remedy, courts have tended to resist enforcing them. One way courts have done so is by finding a waiver of strict performance when a vendor has established a pattern of accepting late payments from the purchaser. To reinstate strict performance, the vendor must notify the purchaser of her intent to do so and must allow the purchaser a reasonable time to make up any late payments.

Even if the contract provides for forfeiture in the event of default, the vendor may not reclaim the land because she has waived strict performance, as explained above.

If the vendor pursues forfeiture, the purchaser would not be liable for damages. It is commonly held that the vendor who elects to pursue a forfeiture cannot also bring an action for damages or for specific performance. If the vendor chooses the forfeiture remedy, she must forgo all others.

48
Q

“Power of sale” provisions generally apply to what type of security interest in land?

A
Promissory notes

B
Mortgages

C
Deeds of trust

A

CORRECT ANSWER: C. Deeds of trust.

“Power of sale” provisions generally apply to deeds of trust. A deed of trust is a security interest in land by which the debtor (i.e., the trustor) transfers title to the land to a third party (i.e., the trustee), such as the lender’s lawyer or a title insurance company, acting on behalf of the lender (i.e., the beneficiary). In the event of default, the lender instructs the trustee to foreclose the deed of trust by selling the property. Many states allow the sale to be either judicial (as with a mortgage) or nonjudicial, under a “power of sale” clause that authorizes the trustee to advertise, give appropriate notices, and conduct the sale personally. The nonjudicial sale is often permitted with deeds of trust but not with mortgages; however, a few states recognize that an effective power of sale can be granted in a mortgage.

Promissory notes are not security interests in land. A security interest operates to secure some other obligation, usually a promise to repay a loan, which is represented by a promissory note.

49
Q

When a mortgagee transfers a promissory note, which of the following is not required for the transferee to become a holder in due course?

A
The transferee must take the note in good faith.

B
The transferee must pay value for the note.

C
The note must state that the transferee takes free of personal defenses.

D
The note must be payable to bearer or to the order of the named payee.

A

CORRECT ANSWER: C. The note must state that the transferee takes free of personal defenses.

When a mortgagee transfers a promissory note, the note does NOT have to state that the holder takes free of personal defenses for the transferee to become a holder in due course. Personal defenses are most defenses available in a standard contract action. The primary benefit of holder in due course status is that a holder in due course takes the note free of any personal defenses that the maker might raise—this need not be reserved in the note itself. A holder in due course is subject only to “real” defenses, such as infancy, duress, and illegality.

When a mortgagee transfers a promissory note, for the transferee to become a holder in due course: 1. The note must be negotiable in form—i.e., the note must be payable to bearer or to the order of the named payee and must contain a promise to pay a fixed amount of money and no other promises, except that it may contain an acceleration clause and an attorneys’ fee clause; 2. If there is a named payee, she must have indorsed (i.e., signed) the note; 3. The note must be delivered to the transferee; and 4. The transferee must pay value for the note and take the note in good faith, without notice that the note is overdue or has been dishonored, or that the maker has any defense to the duty to pay it.

50
Q

A defendant was one of a group of persons who were engaged in a demonstration against the discriminatory practices of a private club. During the demonstration, the defendant threw a bomb containing highly toxic gas through the window of the club. At the time he threw the bomb, he knew that the club’s president was inside the building. Unbeknownst to the defendant, the club’s treasurer was also inside. Each inhaled some gas and were injured, but neither died. The defendant is charged with attempted murder of both the club’s president and treasurer. At trial, the defendant testified that the reason he threw the bomb was that he wanted to make sure that nobody would be able to use the club, and that he did not intend to hurt anyone.

Presuming that the jury believes the defendant, he can be convicted of the attempted murder of:

A
The club’s president.

B
The club’s treasurer.

C
Both the club’s president and the club’s treasurer.

D
Neither the club’s president nor the club’s treasurer.

A

CORRECT ANSWER: D. Neither the club’s president nor the club’s treasurer.

The defendant cannot be convicted of the attempted murder of either the club’s president or treasurer. All attempts require the specific intent to commit the crime. Thus, the defendant must have had the specific intent to kill the president and the treasurer, despite the fact that a defendant may be guilty of common law murder on the showing that the defendant acted with a high degree of recklessness. As a result, if the jury believes that the defendant had no intent to kill either the club’s president or treasurer, he cannot be convicted of attempted murder of either. Thus, (D) is correct, and (A), (B), and (C) are wrong.

51
Q

As a wife was organizing files on the family computer, she discovered emails and photos showing that her husband was having an affair with his secretary. The wife was incensed, so she went to a drawer where she knew her husband kept his handgun and, assuming it was loaded because he always told her that it was, headed off to her husband’s office with the intent to do away with his secretary. When the wife arrived, she burst into the office, pulled the gun out of her purse, and pointed it at the secretary. However, when the wife pulled the trigger, nothing happened, because the handgun contained no ammunition.

The jurisdiction’s criminal code defines assault as “(1) an attempt to commit a battery; or (2) the intentional creation of a reasonable apprehension in the mind of the victim of imminent bodily harm.” The code uses the common law definitions of homicide crimes.

Of which of the following crimes could the wife be convicted?

A
Assault and attempted manslaughter, but not attempted murder.

B
Attempted murder only.

C
Assault and either attempted murder or attempted manslaughter.

D
Attempted murder or attempted manslaughter, but not assault.

A

CORRECT ANSWER: C. Assault and either attempted murder or attempted manslaughter.

C is Correct. The wife could be convicted of assault and either attempted murder or attempted manslaughter. To be liable for either attempted murder or attempted manslaughter, the defendant must have acted with the intent to kill and have committed an act beyond mere preparation for the offense. The fact that it is not possible to complete the intended offense (factual impossibility) is not a defense to liability for attempt. Here, the wife clearly intended to kill the secretary and did everything in her power to carry out the killing. The fact that the gun was not loaded is no defense. (B) and (D) are wrong because the wife could also be convicted of assault. Assault is either: (i) an attempt to commit a battery; or (ii) the intentional creation (other than by mere words) of a reasonable apprehension in the mind of the victim of imminent bodily harm. Here, the wife has committed both types of assault: She attempted to commit a battery against the secretary and intentionally placed her in fear of imminent bodily harm. Although she could not be convicted of the first type of assault if she were also convicted of attempted murder or attempted manslaughter (because that type of assault is a lesser-included offense that merges into the greater offense), she could be convicted of the second type of assault (it does not merge because it contains elements not encompassed by attempted murder or attempted manslaughter). (A) is wrong because the wife could be convicted of attempted murder instead of attempted manslaughter if she cannot establish adequate provocation. An intentional killing can be reduced from murder to voluntary manslaughter if (i) there exists a provocation that would arouse sudden and intense passion in the mind of an ordinary person so as to cause her to lose self-control; (ii) the defendant in fact was provoked; (iii) there was insufficient time for the passions of a reasonable person to cool; and (iv) the defendant in fact did not cool off between the provocation and the killing. Many common law courts recognized the existence of provocation in only two instances: exposure to a threat of deadly force and discovery of one’s spouse in bed with another person. Furthermore, some provocations, such as “mere words,” were defined as inadequate provocation as a matter of law. Modern courts tend to be more reluctant to take such cases from juries and are more likely to submit to the jury the question of whether “mere words” or similar matters constitute adequate provocation. These principles apply in this case even though the wife’s liability is only for attempted murder or manslaughter rather than the completed offense. Here, the wife’s discovery of the emails and photographs might be sufficient to make it a jury question as to whether adequate provocation existed. However, the jury might still conclude that the provocation was not sufficient and find the wife liable for attempted murder rather than attempted manslaughter.

52
Q

A young couple agreed to break into an elderly woman’s house to steal a painting, but they were arrested shortly after they pried open the house’s back door. Both were charged with conspiracy to commit larceny, among other crimes. At her trial, the woman testified that she suspected the man of being a thief, that she agreed to the plan in order to catch him, and that she had made an anonymous telephone call to the police alerting them to the crime to enable them to make an arrest “in the act.” The jurisdiction follows common law conspiracy rules.

As to a charge of conspiracy to commit larceny, if the jury believes the woman, it should find her:

A
Not guilty, because she did not intend to steal.

B
Not guilty, because she prevented the theft from occurring.

C
Guilty, because there was an agreement.

D
Guilty, because she is not a police officer, and thus cannot claim any privilege of apprehending criminals.

A

CORRECT ANSWER: Not guilty, because she did not intend to steal.

The jury should find the woman not guilty of conspiracy because she did not intend to steal the painting. A conspiracy is a combination of or an agreement between two or more persons to accomplish some criminal or unlawful purpose, or to accomplish a lawful purpose by unlawful means. The elements of conspiracy are: (i) an agreement between two or more persons; (ii) the intent to enter into an agreement; and (iii) the intent to achieve the objective of the agreement. Here, if the jury believes that the woman agreed to the plan in order to have the man arrested, she must be found not guilty, given that she would not have had the intent to achieve the objective of the conspiracy. Thus, (A) is correct. (C) is wrong because there was an agreement but no intent on the part of the woman. (B) is wrong because, at common law, withdrawal is not a defense to a conspiracy. Thus, her act of prevention would not have excused the conspiracy if she had the proper mental state. (D) is wrong because the privilege to apprehend criminals is not an issue here.

53
Q

A bartender promised to pay her friend $100 if he would take her television and stereo from her house, so that she could report the items as being stolen to collect a settlement from the insurance company. Although the friend had visited the bartender’s house on two previous occasions, he mistakenly broke into and entered her neighbor’s house and took the neighbor’s television and stereo. When he returned to the bar where the bartender was waiting for him, both were arrested by the police.

If the bartender and her friend are tried for conspiracy, the court will find them:

A
Not guilty, because the friend failed to take the bartender’s property.

B
Not guilty, because the friend, being in the wrong house, could not take the bartender’s property.

C
Guilty, because they actually took the neighbor’s property.

D
Guilty, because they intentionally agreed to defraud the insurance company.

A

CORRECT ANSWER. D. Guilty, because they intentionally agreed to defraud the insurance company.

A conspiracy requires an agreement between two or more people to accomplish an unlawful act or objective. The parties must intend to enter into the agreement and intend to achieve the objective of the agreement. The bartender and her friend agreed to defraud the insurance company and had the requisite intent. They also committed an overt act in furtherance of that intent. Thus, they can be convicted of conspiracy. (A) is wrong because the crime is complete upon agreement with the requisite intent; the objective does not have to be accomplished. (B) is wrong because impossibility is not a defense to conspiracy. (C) is wrong. Taking the neighbor’s property could make them guilty of some other crime, but it is not necessary to convict them of conspiracy.

54
Q

A defendant is on trial for murder. The only evidence linking the defendant to the crime is some blood found at the scene. The lead detective testifies that an officer took a vial containing a blood sample that had been retrieved by a crime scene technician and drove off with it. The officer is now dead. Next, the prosecution presents as a witness a crime lab chemist. The chemist will testify that he took a vial of blood that contained a label identifying it as having been retrieved from the subject crime scene, and that he performed tests that established a match between that blood and a blood sample taken from the defendant.

Is the testimony of the chemist admissible?

A Yes, because there has been proper authentication.

B. Yes, because the chemist qualifies as an expert witness.

C. No, because there is insufficient evidence of chain of custody.

D No, because he did not take the original blood sample at the scene of the crime.

A

CORRECT ANSWER: C. No, because there is insufficient evidence of chain of custody.

The testimony is inadmissible because it has not been shown what happened to the blood between the time the officer took it and the time the chemist examined it. Real evidence presents an object in issue directly to the trier of fact. One of the general requirements for admissibility of real evidence is that it be authenticated; i.e., that it be identified as being what its proponent claims it is. If the evidence is of a type that is likely to be confused or can be easily tampered with, the proponent of the object must present evidence of chain of custody. The proponent must show that the object has been held in a substantially unbroken chain of possession. It is not necessary to negate all possibilities of substitution or tampering; rather, what is required is to show adherence to some system of identification and custody. Here, the proponent of the blood sample (the prosecution) has not shown what the officer did with it after leaving the crime scene. There is no showing that the vial was placed directly in a properly secured area so as to diminish the possibility of tampering. In short, it has not been demonstrated that there was adherence to some defined system of identification and custody. In the absence of a substantially unbroken chain of custody, the evidence is inadmissible for lack of proper authentication, and (A) is incorrect. (B) is incorrect. Although it may be true that the chemist qualifies as an expert witness (i.e., he has special knowledge, skill, experience, training, or education sufficient to qualify him as an expert on the subject to which his testimony relates), the fact remains that his testimony is inadmissible, as explained above. (D) is incorrect because the chemist would be permitted to testify to the results of the blood comparisons if there were proper authentication of the blood taken from the crime scene. There is no need for the chemist to have taken the original blood sample himself.

55
Q

A cattle-producing state adopted a statute requiring any food service business operating in the state to serve beef raised in the United States. A licensed hot dog vendor who worked at a football field within the state and who had been buying hot dogs made with foreign beef for the past several years estimated that switching to an all-beef hot dog made from United States beef would reduce his profits by 10%. An attorney hired by the vendor to challenge the statute discovered during research into the case that most of the footballs used at the football field at which the vendor worked were made of foreign leather.

Which of the following grounds is the vendor’s best argument against the constitutionality of the state statute?

A. The statute burdens foreign commerce.

B. The statute violates equal protection guarantees because it is not rational to prohibit the sale of foreign beef but not foreign leather.

C. The statute substantially interferes with the vendor’s right to earn a living under the Privileges or Immunities Clause of the Fourteenth Amendment.

D. The statute constitutes a taking without due process of law.

A

CORRECT ANSWER: A. The statute burdens foreign commerce.

The best argument against the constitutionality of the state statute is that it burdens foreign commerce. For all practical purposes, the power to regulate foreign commerce lies exclusively with Congress. Therefore, a state that adopts legislation requiring private vendors to favor United States products over foreign products, as the state did here, may be acting outside the scope of its powers. (B) is incorrect because the statute is a rational method of protecting local beef interests. The rational basis standard applies when an economic law, such as the one here, is challenged on equal protection grounds. Under the standard, the Supreme Court will usually defer to a legislature’s decision that the law is rational notwithstanding the fact that the statute is underinclusive. In other words, the law need not address all of the problems that prompted its passage; it will be upheld even if it is only a “first step” toward a legitimate goal. Here, prohibiting the use of foreign beef appears to be a rational method of protecting state beef raisers. Thus, it is irrelevant that the statute is underinclusive in that it allows the use of both United States beef and foreign leather. (C) is incorrect because the right to earn a living is not a privilege under the Fourteenth Amendment, which protects against infringement of rights of national citizenship, such as the right to petition Congress for redress. (Neither would the statute violate the Privileges and Immunities Clause of Article IV, because that clause only prohibits discrimination against citizens of other states and the statute here treats citizens of all states the same.) (D) is incorrect because the vendor had all of the process that was due him. Because the government action here was a general act and not an individualized adjudication, the vendor had no right to an individual hearing; the normal procedure for adopting a statute is all the process that is due.

56
Q

While working on a construction project, a plaintiff was injured when a heavy object struck his knee. Although the plaintiff was fully compensated for his injuries at the time of the incident, he now seeks disability payments from the construction company because he has developed arthritis in the same knee. The construction company claims that the arthritis has nothing to do with the plaintiff’s on-the-job injury and refuses to pay him disability money. The plaintiff sues. A doctor takes the stand to testify for the plaintiff. He is qualified as an expert witness and during direct examination states that in his opinion the blow to the plaintiff’s knee caused his arthritis. On cross-examination, the construction company’s attorney produces a treatise on arthritis and asks the doctor if the treatise is considered to be authoritative. The doctor responds that the treatise is a standard authority in the field, but that he did not rely on it in forming his professional opinion regarding the plaintiff’s condition. The attorney then seeks to introduce into evidence a statement in the treatise that “the idea that arthritis can be caused by a single traumatic event is purely folklore, although it is widely believed by the ignorant who have no scientific basis for their beliefs.” The plaintiff’s attorney objects.

How should the court rule on the admissibility of the statement from the treatise?

A. Admissible, but only for the purpose of impeaching the doctor’s testimony.

B. Admissible, but only as substantive evidence.

C. Admissible, both as substantive evidence and for purposes of impeaching the doctor.

D. Inadmissible.

A

CORRECT ANSWER: C. Admissible, both as substantive evidence and for purposes of impeaching the doctor.

The statement from the treatise is admissible to impeach and as substantive evidence. Under the Federal Rules, learned treatises can be used either for impeachment or as substantive evidence. One way the credibility of an expert witness may be attacked is by cross-examining him as to his general knowledge of the field in which he is claiming to be an expert. This can be done by cross-examining the expert on statements contained in any scientific publication that is established as reliable authority. Reliability of a publication may be established by: (i) the direct testimony or cross-examination admission of the expert, (ii) the testimony of another expert, or (iii) judicial notice. The Federal Rules recognize an exception to the hearsay rule for learned treatises and admit them as substantive evidence if: (i) the expert is on the stand and it is called to his attention, and (ii) it is established as reliable authority (see above). The doctor has admitted on cross-examination that the treatise is authoritative in the field. Thus, the attorney may use the statement in the treatise to attack the doctor’s general knowledge of the field of arthritis by showing that the doctor’s opinion that the blow to the plaintiff’s knee caused his arthritis is considered to be ignorant and unfounded in the text of the treatise. As noted above, such an attack on the doctor’s general knowledge of the field is a proper means of impeaching his credibility. In addition, pursuant to the Federal Rules, the statement may be read into the record as substantive evidence (i.e., as a means of proving that the plaintiff’s arthritis could not have been caused by a single traumatic event, such as the blow to his knee). The statement may be used as substantive evidence because it has been brought to the attention of the doctor during cross-examination and he established it as a reliable authority, and it will be read into evidence while he is on the stand. (A), which reflects the traditional view, is incorrect because the Federal Rules permit the use of the statement in the treatise as substantive evidence. (B) is incorrect because it precludes use of the statement for impeachment purposes. (D) is incorrect because it would not allow introduction of the statement for either impeachment or substantive evidentiary purposes, and thus it is an incorrect statement of the law.

57
Q

The owner of a factory that uses widgets in making its product and a widget maker entered into negotiations over the telephone and, after a time, reached a general understanding that the factory owner would buy widgets from the widget maker. Following their conversation, the widget maker sent the factory owner a contract, which he had already signed, agreeing to sell 1,000 widgets to the factory owner for a total contract price of $10,000. Upon receipt of the contract in the mail, the factory owner signed the contract and deposited an envelope containing the contract in the mailbox located in front of his workplace.

Before the widget maker received the contract, the factory owner had a change of heart. He telephoned the widget maker and told him that he could not afford to buy the widgets he had ordered, and he was “not interested in that contract we talked about.” The widget maker replied, “That’s all right, I understand. Maybe we can do business some other time.” The next day, the signed contract was delivered to the widget maker’s office. The widget maker, also having had a change of mind, decided that he wanted to enforce the contract.

Is the contract enforceable against the factory owner?

A. Yes, because the acceptance occurred prior to rejection.

B. Yes, because of the parol evidence rule.

C. No, because the offer to rescind was accepted and that discharged the original contract.

D. No, because the rejection by telephone voided the acceptance by mail.

A

CORRECT ANSWER: A. Yes, because the acceptance occurred prior to rejection.

The contract is enforceable because the mailbox rule applies here. Acceptance by mail creates a contract at the moment of posting, properly addressed and stamped, unless the offer stipulates that acceptance is not effective until received, or an option contract is involved. If the offeree sends an acceptance and then rejects the offer, the mailbox rule applies; i.e., a contract is created upon dispatch of the acceptance. Because no option contract is involved here, and the widget maker’s offer did not state that the factory owner’s acceptance would be effective only when received, his acceptance was effective the moment he placed the envelope containing the contract in the mailbox. His attempt to reject occurred after acceptance took place. Thus, a valid contract was formed and the widget maker may enforce it. (B) is incorrect because nothing in the parol evidence rule would serve to validate the contract. Ostensibly, this choice implies that there is a contract because the parol evidence rule will prevent the factory owner from introducing the oral rescission. However, as discussed below, the rescission is ineffective because there was no meeting of the minds. The parol evidence rule would not prevent introduction of the rescission if it were otherwise valid. The parol evidence rule of the UCC merely prohibits a party to a goods contract from contradicting an integrated writing with evidence of any prior agreement or contemporaneous oral agreement. Subsequent agreements such as the attempted rescission here can be introduced. Therefore, the parol evidence rule does not serve to validate the contract here. (C) is incorrect because there is no “meeting of the minds” concerning the rescission. A contract may be discharged by an express agreement between the parties to rescind; the agreement to rescind is itself a binding contract. Because the widget maker did not know that the factory owner had accepted the contract, his statement that “that’s all right” cannot be construed as acceptance of the factory owner’s offer to rescind. Therefore, a contract to rescind was not formed. (D) is incorrect because the telephone rejection did not void the acceptance by mail. As discussed above, if the offeree sends an acceptance first, followed by a rejection, the mailbox rule applies; i.e., a contract is created upon dispatch of the acceptance. Because the factory owner’s telephone rejection took place after his acceptance by mail, his acceptance was effective and a contract was created when the letter was mailed. While an offeree will be estopped from enforcing the contract if the offeror receives the rejection first and changes his position in reliance on it, the widget maker is the one wanting to enforce the contract here.

58
Q

After reaching an oral agreement on the terms of representation, a law firm, at its clients’ behest, instituted a class action lawsuit against a tobacco company for $100 million. Prior to signing the written contract outlining the parties’ rights and responsibilities, including the fee arrangement, the firm’s senior partner told the clients’ representative in a moment of goodwill and generosity that if they won or the tobacco company settled, he would turn over half of the attorneys’ fees in the case to a particular nonprofit group that funds research on lung cancer and other respiratory illnesses. After the law firm won the case and collected its fee of $33 million, it had second thoughts about turning over half of it to the nonprofit group.

If the nonprofit group sues the law firm in an attempt to collect the $16.5 million, which of the following is the law firm’s best defense to such action?

A. The promise to turn over half of the attorneys’ fees was discharged by innovation.

B. A partial assignment (in favor of the nonprofit group) is ineffective.

C. The promise to turn over half of the attorneys’ fees was not in writing.

D. The law firm was simply attempting to confer a gift upon the nonprofit group.

A

CORRECT ANSWER: D. The law firm was simply attempting to confer a gift upon the nonprofit

The nonprofit group did not give consideration to the law firm in return for the law firm’s promise to turn over half of its attorneys’ fees to the group in the event it won or settled the class action suit. Thus, the law firm’s promise was gratuitous; i.e., it was simply attempting to confer a gift upon the nonprofit group, and the group could not compel the law firm to turn over the money. (A) is incorrect because there was no novation and, hence, no discharge of the promise. A novation substitutes a new party for an original party to a contract, and in the process discharges the old contract and creates a new one. Here, the parties to the contract (the law firm and the clients) did not change when the law firm, through its senior partner, made its gratuitous promise. (B) is incorrect because partial assignments are effective—an assignor may transfer some rights under the contract and retain others. (C) is incorrect because the promise did not fall within the Statute of Frauds, which requires a writing signed by the party to be bound. Although the amount in question was $500 or more, the promise was not for the sale of goods. Moreover, this was not a promise that, by its terms, could not be performed within a year—there was a possibility of completion within one year, for example, if the parties settled. Thus, the promise was not unenforceable due to the lack of a written agreement.

59
Q

A new federal law prohibited the use of various pesticides in areas with a certain population density near navigable waters. A city located in the southeastern United States was plagued by a sharp increase in disease-carrying mosquitoes. The city’s board of health recommended that all residential areas be sprayed with a pesticide proven to be highly effective against mosquitoes. Despite the fact that the federal law would prohibit use of that pesticide in these areas, the city council passed an ordinance adopting the board of health plan, relying on the opinions of several independent experts that the health benefits of reducing the mosquito population outweighed the risks of spraying. An environmentally minded citizen of the city brought an action in federal court challenging the ordinance.

Assuming that the citizen has standing, is the court likely to find the ordinance valid?

A. Yes, because pursuant to the police power, cities have a compelling interest in laws designed to protect the health, safety, and welfare of their citizens.

B. Yes, because controlling health hazards is an integral governmental function.

C. No, because it is superseded by the power of Congress to adopt laws to protect the health, safety, and welfare of citizens.

D. No, because it conflicts with a federal law that Congress had the power to make under the Commerce Clause.

A

CORRECT ANSWER: D. No, because it conflicts with a federal law that Congress had the power to make under the Commerce Clause.

Congress’s power to regulate commerce has been construed broadly, so that it may regulate any activity, local or interstate, that either in itself or in combination with other activities has a substantial economic effect on interstate commerce. If Congress has determined that the use of chemical pesticides and their runoff into waterways (which are channels of interstate commerce) will have an overall detrimental impact on the environment, this determination will be sufficient in this case to satisfy the standards established by the Supreme Court. Therefore, the law probably is a valid exercise of the commerce power. Any state or local action that conflicts with a valid act of Congress is invalid under the Supremacy Clause. (A) is incorrect because while the police power (the power to adopt regulations for the health, safety, and welfare of citizens) belongs to the states, a police power regulation that conflicts with a federal law is invalid under the Supremacy Clause. (B) is incorrect because state and local government activities may be regulated by a general law that applies to both the public and private sectors, even if the regulation affects integral governmental functions, as long as there is a constitutional basis for the law. (C) is incorrect because Congress does not have a general “police power” to adopt laws on health and safety. The laws that Congress has passed banning activities that it has deemed harmful to public health have been based on its power to regulate interstate commerce.

60
Q

A critically acclaimed movie that had received a number of awards opened in a small town. The film had portrayals of nudity and scenes involving sexuality, but its advertising was very tasteful and concentrated on its critical acclaim and its receipt of seven Academy Award nominations. Nevertheless, when the movie opened in the small town, there was a public outcry against it, including picketing. The town, which had been founded in the late nineteenth century by a fundamentalist religious group, remained very conservative and highly religious, and was the only community in the state where a consensus of the community would find the movie to be obscene. The town prosecutor went to the local court seeking an injunction to halt the showing of the movie. The theater owner refused to voluntarily stop showing the film and appeared in court to defend against the proposed injunction.

What is the owner’s best defense?

A. The proper “community standards” should be those of the entire state rather than of the town.

B. The film has some redeeming social value.

C. The Establishment Clause of the First Amendment prevents the state from enforcing a particular set of religious beliefs.

D. The film has proven artistic merit.

A

CORRECT ANSWER: D. The film has proven artistic merit.

The theater owner’s best defense is that the film has proven artistic merit. The First Amendment generally protects the right of freedom of speech, and this freedom includes the right to show movies. Thus, to enjoin the showing of the movie here, the city will have to prove that the speech involved is unprotected speech. Obscenity is the category of unprotected speech most relevant here. The Court has defined obscenity as a depiction of sexual conduct that, taken as a whole, by the average person, using contemporary community standards: (i) appeals to the prurient interest in sex; (ii) portrays sex in a patently offensive way; and (iii) using a national, reasonable person standard, does not have serious literary, artistic, political, or scientific value. If the theater owner shows that the film has proven artistic merit, it cannot be held to be obscene because the third element of the above definition will have failed. Thus, (D) is his best argument. (A) is not as good an argument as (D) because the Supreme Court has held that while a statewide community standard may be used, it is not mandatory-a local community standard is sufficient to evaluate whether the film is “patently offensive.” Thus, the town’s community standards would be sufficient. (B) is not as good an argument as (D) because it is not sufficient that there is some redeeming social value; it must have serious redeeming value, as indicated above. For example, it would not be sufficient that an otherwise obscene movie included short tips on the importance of brushing teeth. (C) is not a good argument. The Establishment Clause forbids the government from adopting a law or program that establishes religion. It is inapplicable here because the town is not trying to enforce a particular set of religious views; rather, it is trying to prohibit obscenity. The Supreme Court has held that the government has a legitimate interest in prohibiting obscenity, and the fact that this happens to coincide with the beliefs of a particular religious group does not render such bans void.

61
Q

A defendant was convicted of burglary after the prosecution established at a jury trial that the defendant broke into the victim’s house at night by cutting open a window screen and climbing into a bedroom. The evidence also established that the defendant was frightened into leaping out the window when the victim pulled a gun from beneath her pillow. The defendant testified in his own defense, stating that he entered the house merely to use the telephone. His lawyer thus argued that the defendant lacked the required intent for burglary. The trial court, after instructing the jury on the elements of burglary, said, “If you find that by a fair preponderance of the evidence the defendant has shown that he intended to use the telephone when he entered the victim’s home, then you must find him not guilty.”

If the defendant appeals his conviction, will he likely obtain a reversal?

A
Yes, because the trial court’s instruction permitted the jury to use a preponderance standard rather than beyond a reasonable doubt standard.

B
Yes, because the trial court’s instruction placed the burden of proof on the defendant.

C
No, because any error in instructions was harmless because it is more likely than not that the jury would have convicted him anyway.

D
No, because the trial court is permitted to comment upon the evidence.

A

CORRECT ANSWER: B. Yes, because the trial court’s instruction placed the burden of proof on the defendant.

B is Correct. The defendant will likely obtain a reversal. Burglary is the breaking and entering of the dwelling house of another in the nighttime with the intent to commit a felony inside the house. The prosecution must prove every element of the offense, including intent, beyond a reasonable doubt. Given that the trial court’s instructions placed the burden of proving lack of intent on the defendant, they were in error. Thus, (B) is correct, and (D) is incorrect. (A) is incorrect because it focuses on the standard of proof, when the burden of proof is the issue. (C) is incorrect because it is impossible to make the harmless error analysis without knowing more about the state of the evidence, and the test for constitutional error is “harmless beyond a reasonable doubt,” not by a preponderance, as suggested by the answer.

62
Q

A landowner leased 150 acres of farmland to a produce company for 15 years. The produce company used the land for crops along with several other contiguous acres that it owned or leased. About four years into the lease, the state condemned a portion of the leased property because it intended to build a highway. As a result, too little property remained for the produce company to profitably farm, although there still existed the farmhouse on the property, which was being used by one of its foremen. The produce company gave the landowner 30 days’ written notice that it considered the lease to have been terminated because of the condemnation.

In a suit for breach of contract, is the landowner likely to win?

A. No, because the condemnation made it economically undesirable for the produce company to continue to lease the property.

B. No, because when there is a condemnation, the tenant’s obligation to pay rent is extinguished.

C. Yes, because the produce company can still use the farmhouse, and the rental value would be adjusted accordingly.

D. Yes, because the condemnation did not affect the produce company’s obligation to pay the full rental price, although it is entitled to share in the condemnation award.

A

CORRECT ANSWER: D. Yes, because the condemnation did not affect the produce company’s obligation to pay the full rental price, although it is entitled to share in the condemnation award.

The landowner probably will win in a breach of contract suit. In partial condemnation cases, the landlord-tenant relationship continues, as does the tenant’s obligation to pay the entire rent for the remaining period of the lease. The tenant is, however, entitled to share in the condemnation award to the extent that the condemnation affected the tenant’s rights under the lease. Therefore, (B) and (C) are incorrect. (A) is not correct because the law of landlord and tenant traditionally refuses to recognize frustration of purpose as grounds for termination of a lease.

63
Q

The defendant robbed a bank and fled in a getaway car driven by an accomplice, not realizing that one of the bundles of money he took had the serial numbers recorded and had a tiny tracking device attached to the wrapper. The bank’s security consultant obtained portable tracking equipment and was able to trace the bundle of money to the defendant’s house. The police were notified and they arrived at the defendant’s house a few hours after the robbery. They knocked on the door, announced their presence, and saw someone matching the description of the robber in the hallway. They entered and arrested the defendant, and then conducted a protective sweep of the house for the accomplice, who they believed had a gun. They did not find him, but while checking a closet, they discovered several of the bundles of money from the bank and a gun the defendant had used in the robbery. The police also discovered two clear plastic bags of what appeared to be marijuana sitting on top of a dresser. They seized the money, the gun, and the two bags. Later testing confirmed that the substance in the bags was marijuana.

The defendant was charged with the bank robbery and with possession of the marijuana. At a preliminary hearing, he moves to suppress introduction of the money, gun, and marijuana.

How should the court rule?

A. Grant the motion as to the marijuana but not as to the money or the gun because the money and gun were found as a result of the protective sweep for the defendant’s accomplice.

B. Grant the motion as to the money and the gun but not as to the marijuana because the bags containing the marijuana were clearly visible on the dresser during the search.

C. Grant the motion as to all of the evidence seized.

D. Deny the motion as to all of the evidence seized.

A

CORRECT ANSWER: C. Grant the motion as to all of the evidence seized.

The court should suppress all of the evidence because it was the fruit of an unconstitutional arrest. As a general rule, the police must have an arrest warrant to effect an arrest of an individual in his own home. There is no general “emergency” exception to the warrant requirement. While police officers in hot pursuit of a fleeing felon or trying to prevent the destruction of evidence may sometimes make a warrantless search and seizure, the burden is on the government to show that one of those exceptions applies. Here, the police did not arrive at the defendant’s house in hot pursuit of the defendant, and there was no indication that the defendant might be destroying the money or other evidence; i.e., there were no circumstances precluding them from keeping the house under surveillance while they obtained a warrant. Hence, the arrest was unconstitutional. Because an arrest constitutes a seizure under the Fourth Amendment, the exclusionary rule applies, and evidence that is the fruit of the unconstitutional arrest may not be used against the defendant at trial. Here, all of the evidence was seized without a warrant, and none of the other exceptions to the warrant requirement are applicable. While the protective sweep that turned up the money and gun probably would have been within the bounds of a search incident to an arrest because the police had reason to believe an armed accomplice was present, the arrest in violation of the Fourth Amendment makes the search unlawful. Similarly, while the bags of marijuana were discovered in plain view, the police have to be legitimately on the premises for that exception to apply. Thus, (C) is correct; (A), (B), and (D) are incorrect.

64
Q

Small, prolific mussels called zebra mussels were first introduced into the Great Lakes by a foreign cargo ship. They became a serious problem because they attached themselves to smooth, hard surfaces, and often clogged water intake pipes. Congress determined that zebra mussels posed a great threat to the economic welfare of the Great Lakes region and passed a statute requiring all Great Lakes water intakes to be coated with a special chemical compound that repels zebra mussels. Studies by biologists at a major state university showed that while the special chemical compound that the federal government has required was effective, it also was toxic to other aquatic life. The biologists recommended that Great Lakes intake pipes be coated with a less toxic and less expensive copper-based paint. On the basis of those studies and the recommendation, three Great Lakes states adopted laws permitting municipal water districts to coat their intake pipes with copper paint.

Can municipalities using copper-based paint on their intake pipes successfully be prosecuted for violating the federal law?

A. No, because the Tenth Amendment prevents Congress from interfering with integral government functions.

B. No, because the municipalities are taking effective steps to combat zebra mussels in compliance with the spirit and purpose of the federal law.

C. Yes, because Congress is in a better position to regulate the entire Great Lakes region than the individual states.

D. Yes, because Congress may adopt laws regulating navigable waters.

A

CORRECT ANSWER: D. Yes, because Congress may adopt laws regulating navigable waters.

The cities can be prosecuted because state or local government action that conflicts with valid federal laws is invalid under the Supremacy Clause. The federal law here could be found valid as an exercise of the commerce power (Congress can regulate any activity that either in itself or in combination with other activities has a substantial economic effect on interstate commerce) or under the admiralty power (Congress can regulate all navigable waterways). The action of the municipalities directly conflicts with the directives of the federal law and can therefore be stopped. (B) is incorrect because the fact that the copper paint may be as effective as the special compound does not change the result. The action by the municipalities can be prohibited under the Supremacy Clause. (A) is incorrect because for regulations that apply to both the public sector and the private sector, the Supreme Court has held that states’ Tenth Amendment rights are best protected by the states’ representation in Congress; hence, the Tenth Amendment is not a likely ground for striking this federal legislation because it is not directed only at state or local governments. (C) is incorrect because it is irrelevant; the federal law is superior to the states’ laws because it is within Congress’s power, not because Congress is in a better position than the states to adopt the legislation involved.

65
Q

While driving in a city in State A, citizens of State B and State C simultaneously struck a pedestrian, who was a citizen of State A. The pedestrian properly sued both drivers in a federal district court in State A, seeking $100,000 based on negligence.

If the State B driver files a cross-claim for negligence against the State C driver to recover damages for their injuries in the accident, can the driver join with that cross-claim an unrelated claim that they have against the State C driver?

A. No, because defendants cannot add claims against co-defendants that are not related to the original claims asserted by the plaintiff.

B. No, because the court does not have supplemental jurisdiction over the State B driver’s unrelated claim against the State C driver.

C. Yes, the State B driver must assert the unrelated claim or he will be foreclosed from asserting it later.

D. Yes, the State B driver may join their unrelated claim with his negligence claim against the State C driver, but they are not required to do so.

A

CORRECT ANSWER: D. D. Yes, the State B driver may join their unrelated claim with his negligence claim against the State C driver, but they are not required to do so.

The State B driver can join their unrelated claim with their negligence claim, but they are not required to do so. As a general rule, a party may assert a cross-claim against a co-party only if the cross-claim arises from the same transaction or occurrence as the original action or a counterclaim. However, once the party has asserted such a cross-claim, they may join with it any other claim that they has against the same party. Here, the State B driver’s negligence claim would related to the pedestrian’s negligence claim and is thus a proper cross-claim. Thereafter, the State B driver may join any unrelated claims they have against the State C driver, so long as there is subject matter jurisdiction. (And there would be subject matter jurisdiction here under the court’s diversity jurisdiction.) Thus, the unrelated claim also can be asserted in this case. (A) is therefore incorrect. (B) is incorrect because, as stated, the court would have diversity jurisdiction over the unrelated claim. (C) is incorrect because the unrelated claim is not a compulsory counterclaim.

66
Q

A woman sued her ex-husband for installments due under their divorce property settlement agreement. The ex-husband defended the suit on the ground that the ex-wife was in breach of the agreement. However, the court granted judgment for the ex-wife. Now, the ex-wife is suing her ex-husband for other installments due under the agreement. The ex-husband raises the defense that the agreement was void and illegal because of fraud perpetrated by his ex-wife at the time the agreement was signed. The ex-wife moves to strike her ex-husband’s defense, claiming issue preclusion.

Should the court grant the motion?

A. No, because the issue was never litigated.

B. No, because judgment on one installment does not bar subsequent actions on other installments.

C. Yes, because the husband should have raised the defense in the first action.

D. Yes, because the issue of fraud is never waived.

A

CORRECT ANSWER: A. No, because the issue was never litigated.

The court should not grant the motion. Generally, issues actually litigated between the parties are binding on them in subsequent actions concerning the same claim. If the second suit involves a different claim, the first judgment may be invoked as to all matters actually litigated and determined in the first action, provided that the findings were essential to the first judgment. Here, the fraud issue was not actually litigated in the first action, and thus it can be raised with respect to the later installments. (B) is incorrect. The general principle recited in this answer is true for claim preclusion (unless there is an acceleration clause). However, even if claim preclusion does not apply, if the defendant raises the same issues as in the first case, he may be prevented by issue preclusion (collateral estoppel) from re-litigating those issues in that subsequent case. (C) is incorrect. Claim preclusion (res judicata) would bar claims arising out of the same transaction or occurrence. Here, claim preclusion does not apply because a defense, not a claim, is involved, and the second installment is a separate transaction or occurrence. (D) is incorrect because there is no such rule.

67
Q

A backgammon player was upset after losing a match against the club champion. Rushing out of the club, he inadvertently grabbed the champion’s board, which looked very much like his own but which was much more expensive. The player left the backgammon board in the trunk of his car, as was his usual practice. During the night, the car was stolen and along with it, the champion’s expensive backgammon board.

In an action by the champion against the player to recover the value of the backgammon board, is the champion likely to recover?

A. Yes, because when the player took the backgammon board he committed a trespass to the champion’s chattel.

B. Yes, because when the backgammon board was stolen along with the car, the player became liable for conversion of the champion’s chattel.

C. No, because the player believed in good faith that the board was his when he took it from the backgammon club.

D. No, because the backgammon board was lost through no fault of the player’s.

A

CORRECT ANSWER: B. Yes, because when the backgammon board was stolen along with the car, the player became liable for conversion of the champion’s chattel.

The champion will recover from the player for conversion. Conversion consists of (i) an act by defendant interfering with plaintiff’s right of possession in the chattel, (ii) intent to perform the act bringing about the interference with plaintiff’s right of possession, (iii) causation, and (iv) damages—an interference that is serious enough in nature or consequence to warrant that the defendant pay the full value of the chattel. Intent to trespass is not required; intent to do the act of interference with the chattel is sufficient for liability. Therefore, the player was guilty of conversion when he intentionally (i.e., volitionally) took the champion’s board, which resulted in its loss, even though the player did not intend to lose it or even realize that he had taken the property of another. (A) is not the best answer because complete loss of a chattel, permitting the plaintiff to recover its full value, is too serious an interference to be classified a mere trespass. Trespass to chattels consists of: (i) an act by defendant interfering with plaintiff’s right of possession in the chattel, (ii) intent to perform the act bringing about the interference with the plaintiff’s right of possession, (iii) causation, and (iv) damages. Had the champion been able to recover the board, and had he been able to show actual damages during the time of dispossession, he might have been able to recover for trespass to chattels. (C) is wrong because the player’s good faith is irrelevant. Even if the conduct is wholly innocent, liability will attach when the interference with the chattel is serious in nature. (D) is wrong because the fact that the player’s car was stolen does not relieve him of liability. His initial trespassory interference with the champion’s backgammon set was a substantial factor in its complete loss, because it would not have otherwise been in the trunk of his car. Thus, the causation element for conversion is satisfied.

68
Q

In an effort to fix a longstanding rodent problem, a city enacted a statute that, in subsection one, required that all food waste be stored in lidded containers made of plastic or metal and, in subsection two, prohibited the outdoor use of any waste container that did not have a lid. Violation of the statute was considered to be a misdemeanor, and the statute provided for a fine of $50 for each violation. The defendant knew about the statute, as its enactment was well-publicized, but he misread the statute and believed that subsection two applied only to food waste. In the course of cleaning out his home office, the defendant put many old files into cardboard boxes without lids, which he then left on his curb for his trash collector to pick up that same day. The defendant was promptly fined $50.

Did the defendant commit any offense?

A
No, because he had no intent to violate the statute.

B
No, because the mistake of law was reasonable, given that the first subsection only applied to food items.

C
Yes, because the defendant’s mistake of fact that the statute applied only to food items is not a defense.

D
Yes, because the defendant’s mistake of law as to the application of the statute is not a defense.

A

CORRECT ANSWER: D. Yes, because the defendant’s mistake of law as to the application of the statute is not a defense.

The defendant’s mistaken belief that the statute only applied to food items provides no defense to the crime charged. It is not a defense to a crime that the defendant was unaware that his acts were prohibited by the law or that he mistakenly believed that his acts were not prohibited, even if such ignorance or mistake was reasonable. Here, the defendant knew that he was storing garbage in a container that did not have a lid, but was unaware that this constituted a violation of subsection two. Such ignorance of law will not be a defense to the misdemeanor charge, even if such ignorance was reasonable. (A) is incorrect because the crime here appears to be a strict liability crime because it is a regulatory offense with a relatively low penalty; thus, the defendant’s intent to violate the statute is irrelevant. (B) is also incorrect. Even if the defendant reasonably believed the statute applied only to the disposal of food items, his mistake of law would not provide a defense, as explained above. The statute is clear in its terms, and was reasonably available for the collector’s perusal. (C) is incorrect because defendant’s mistake is one of law, not fact.

69
Q

Late one night, a young couple were killed instantly when their car was struck by a speeding truck as the couple’s car crossed an intersection with the light green in their favor. Several weeks later, a burglar awaiting trial on burglary charges asked a jail officer to let him speak with a highway patrol officer. When the highway patrol officer came to the cell, the burglar told him that he was the driver of the truck that had struck the car and had been speeding away from a burglary when the accident occurred.

The burglar was charged with felony murder, on the theory that he had not yet reached a place of temporary safety when the accident occurred. At trial, the prosecution seeks to introduce the burglar’s statements to the highway patrol officer regarding the events of the night of the accident. The burglar’s attorney objects.

Which of the following is the strongest argument for permitting the statements into evidence?

A
The burglar had not been charged in connection with the auto accident at the time the statements were made to the highway patrol officer.

B
The burglar made the statements spontaneously, without inducement or interrogation by the police.

C
The highway patrol officer had no connection with the burglary investigation for which the burglar had been incarcerated.

D
The burglar’s statements were not the product of coercion by the police officers.

A

ANSWER: B. The burglar made the statements spontaneously, without inducement or interrogation by the police.

The best argument is that the burglar made the statements spontaneously. Prior to a suspect’s being charged with a crime, the Fifth Amendment privilege against compelled self-incrimination is the usual basis for ruling on the admissibility of a confession. [Miranda v. Arizona (1966)] Under Miranda, statements made during custodial interrogations are inadmissible unless the defendant is first warned of his right to remain silent and his right to an attorney. Thus, Miranda applies only when the defendant is in custody and only when the defendant’s statements are the result of interrogation. Although almost any words or actions on the part of the police that they should know are reasonably likely to elicit an incriminating response qualify as interrogation, Miranda does not apply to spontaneous statements not made in response to interrogation. Here, the police did nothing to solicit the statement from the burglar; it was spontaneous. Thus, (B) is correct. (A) is incorrect because the defendant need not yet be charged for Miranda rights to apply as long as he is in custody (i.e., not free to leave). Being in jail on another charge (as the burglar was) satisfies the custody requirement. (C) is incorrect because the fact that the officer who took the burglar’s admission had nothing to do with the investigation of the burglary does not alter the rules of Miranda—questioning that is totally unrelated to the matter for which the accused is in custody may still violate the accused’s Miranda rights. (D) is incorrect. Due process requires that a confession be voluntary (i.e., not the product of police coercion). The Miranda rule, however, goes beyond voluntariness. It makes inadmissible all statements obtained without Miranda warnings or without a valid waiver of Miranda rights, not just statements actually coerced by the police.

70
Q

A woman called the police to report that she had been assaulted. She gave the police a detailed description of her attacker, and they picked up a man who matched the description who was found near the site of the alleged attack. The police took the man to the police station and read him his Miranda warnings. The man asked for a public defender to be appointed. Before the public defender arrived, the woman came to the police station and was told there would be a lineup as soon as the suspect’s lawyer arrived. On the way to the viewing room, the woman passed a holding cell where the man was being held. She pointed at him and said loudly, “That’s the man who attacked me!” The man did not respond in any way. The woman later picked the man out of a lineup.

At the trial, if the prosecutor wishes to introduce evidence that the man said nothing when the woman confronted him, would such evidence be admissible?

A
Yes, because it accurately describes how the man responded.

B
Yes, because the man had been read his Miranda warnings and knew that any behavior could be used against him.

C
No, because the man’s right against self-incrimination would be violated if he were required to speak.

D
No, because counsel was not present at the time of the incident.

A

CORRECT ANSWER: C. No, because the man’s right against self-incrimination would be violated if he were required to speak.

Introduction of the proffered evidence would in effect penalize the man for exercising his right to be free from compulsory self-incrimination. Miranda warnings are given to safeguard the Fifth Amendment right to be free from compelled self-incrimination. Prior to interrogation, a person in custody must be clearly informed that he has the right to remain silent and anything he says can be used against him in court. These warnings implicitly assure that silence will carry no penalty. Thus, a prosecutor may not comment on a defendant’s silence after the defendant is arrested and has received the Miranda warnings. To allow the prosecutor to introduce evidence of the man’s silence in the face of the woman’s accusation would run counter to the very purpose of the Miranda warnings, which is to allow the defendant to remain silent without fear of being prejudiced by such silence. The man was no more required to respond to the woman’s accusation than he would have been to an accusation or question coming from the police. The man’s privilege against compelled self-incrimination would be meaningless if he were required to either respond to the woman or have his failure to respond introduced against him. (A) is wrong because the mere fact that evidence is true does not render such evidence admissible. Evidence that runs afoul of some rule of evidentiary exclusion (e.g., a statement that is hearsay not subject to any of the exclusions to the hearsay rule) or that would violate a right of constitutional magnitude is not admissible, regardless of whether it is true. Because evidence of the man’s silence would violate his Fifth Amendment right to be free of compelled self-incrimination, this evidence is not admissible, even though it accurately describes how the man responded. (B) is wrong because it turns the Miranda warnings on their head. The warnings are to apprise the suspect that he has a right to remain silent, and that if he chooses to say something, this statement can be used against him. The warnings do not, as (B) suggests, apprise the suspect that he must respond to questions or accusations, and that a failure to respond can be used against him. As has been explained above, the crux of the Miranda warnings is the right to remain silent, and to be free of coercion to speak at all. (D) is wrong for two reasons. (D) implies that the evidence would be admissible if the man’s lawyer were present at the time of the incident. Regardless of the presence of counsel, admission of evidence as to the man’s silence in the face of the woman’s accusation would violate the man’s privilege against compelled self-incrimination. Also, events had not yet reached a stage at which the man was entitled to counsel. At the time of this incident, the man was waiting to take part in a lineup. The facts indicate that the man had not yet been charged with a crime. The right to counsel applies to post-charge lineups, but not to pre-charge lineups. Note also that the woman’s accusation did not even occur in a lineup setting, but during an inadvertent viewing of the man. Thus, the right to counsel had not yet attached.

71
Q

A defendant was tried in state court for criminal fraud. The jury found the defendant guilty. A state statute provided that a judge could increase the sentence for fraud by up to five years beyond the statutory maximum if the judge found the existence of certain aggravating factors. Based on evidence presented to the jury that found the defendant guilty, the trial judge found that sufficient aggravating factors existed and imposed a sentence on the defendant for five years longer than the statutory maximum.

The defense attorney objected to the sentence. The judge overruled the objection and the defendant appealed his sentence to an appropriate appellate court.

How should the appellate court rule?

A
For the state, because the statute gave the defendant fair warning that the sentence could be increased.

B
For the state, because the jury determined guilt.

C
For the defendant, because the jury did not determine whether the aggravating factors existed.

D
For the defendant, because the special rules for capital offenses do not apply here.

A

CORRECT ANSWER: C. For the defendant, because the jury did not determine whether the aggravating factors existed.

(C) is correct. The Sixth Amendment guarantees the right to a jury trial. When a defendant elects a jury trial, he has a right to have the jury determine all the facts. If the substantive law provides that a sentence may be enhanced beyond the statutory maximum only upon a finding of certain facts (other than prior convictions), those facts must be found to exist by the jury. A determination of their existence by a judge is a denial of the Sixth Amendment right to a jury trial. (A) is incorrect; fair warning of the possible sentence enhancement for purposes of due process is irrelevant to upholding the defendant’s Sixth Amendment right to a jury. (B) is incorrect, because a jury must determine any fact needed to find guilt or to impose an additional sentence. (D) is incorrect; though there are special rules for capital offenses, that issue is irrelevant to the facts presented here.

72
Q

A woman and her boyfriend worked together as pickpockets. The woman approached a man from the front to distract him, holding a small camera and asking him to take a picture, while her boyfriend came up from behind with a knife to slice open the back pocket of the man’s pants and remove his wallet. The man was drunk and believed that the woman had a gun and was trying to rob him, but he was unaware of the boyfriend behind him. The man reached into his back pocket to hand over his wallet and was cut by the boyfriend’s knife as it was slicing through his pocket. The wallet dropped to the ground as the man clutched his hand. The boyfriend picked it up, and he and the woman fled while the man knelt on the ground in pain. The woman was apprehended shortly thereafter and charged with robbery.

Should the woman be found guilty?

Question ID: MC003

A
Yes, because her boyfriend obtained the property by means of force.

B
Yes, because the man believed that the woman would shoot him if he did not give up his wallet.

C
No, because neither the woman nor her boyfriend intended to use force against the man to obtain the property.

D
No, because the man’s belief that the woman was robbing him was unreasonable.

A

CORRECT ANSWER: A. Yes, because her boyfriend obtained the property by means of force

A is Correct. The woman should be found guilty of robbery because her accomplice obtained the wallet by means of force. Robbery consists of (i) a taking (ii) of personal property of another (iii) from the other’s person or presence (iv) by force or intimidation (v) with the intent to permanently deprive him of it. Thus, robbery is basically an aggravated form of larceny in which the taking is accomplished by force or threats of force. The force must be used either to gain possession of the property or to retain possession immediately after such possession has been accomplished, but the defendant need not have intended to use force to complete the crime; the only intent required is the intent to permanently deprive the victim of his property. Here, the woman and her boyfriend had such intent, and they were able to carry out that intent in part because the boyfriend slashed the man’s hand with the knife, incapacitating him. The fact that the boyfriend did not intend to injure the man is irrelevant; hence, (C) is wrong. (B) is wrong because the man’s erroneous belief that he was being threatened does not establish the element of threat or intimidation. The woman’s conduct was merely an attempt to distract the man and did not constitute a threat or intimidation; the fact that the man’s intoxication caused him to believe otherwise does not change that result. (D) is wrong because the unreasonableness of the man’s belief does not change the fact that the woman is liable as an accomplice to the robbery by her boyfriend, because robbery (the use of force) was a foreseeable consequence of the pickpocketing.

73
Q

Criminal statutes in the state define murder as “the unlawful killing of another human being with malice aforethought, either express or implied,” and define voluntary manslaughter as “the unlawful killing of another human being under an extreme emotional disturbance for which there was reasonable explanation or excuse, without express or implied malice aforethought.” Another statute provides that “it shall be an affirmative defense to the charge of murder if the defendant proves by clear and convincing evidence that the defendant was unable to control his actions or conform his conduct to the law.”

The defendant was charged with murder and tried in state court. At trial, he introduced evidence regarding his state of mind at the time of the homicide, including testimony from a psychiatrist. At the conclusion of the case, the court instructed the jury as follows:

“There are two kinds of homicide in our state: murder and manslaughter. The common elements of both are that the homicide be unlawful—i.e., neither justifiable nor excusable—and that it be intentional. Malice aforethought is an indispensable element of the crime of murder. However, if the prosecution can establish that the killing was both intentional and unlawful, malice aforethought may be presumed unless the defendant proves by a preponderance of the evidence that he acted under extreme emotional disturbance for which there was reasonable explanation or excuse, in which case he shall be liable only for voluntary manslaughter, because extreme emotional disturbance is inconsistent with, and negates the existence of, malice aforethought.”

The judge further instructed the jury that it could return an acquittal by reason of insanity “if the defendant established by clear and convincing evidence that he was unable to control his actions or conform his conduct to the law.” The jury found the defendant guilty of murder. The defendant appealed, claiming that his constitutional rights were violated by the court’s instructions as to homicide and by the requirement that he prove insanity by clear and convincing evidence.

Were the defendant’s rights violated?

A
Yes, by the court’s instructions as to homicide, but not the requirement of clear and convincing evidence for insanity.

B
Yes, by the requirement of clear and convincing evidence for insanity, but not the instructions as to homicide.

C
Yes, by both the homicide instructions and the clear and convincing evidence requirement.

D
No, neither the homicide instructions nor the clear and convincing evidence requirement violated the defendant’s rights.

A

CORRECT ANSWER: A. A
Yes, by the court’s instructions as to homicide, but not the requirement of clear and convincing evidence for insanity.

The defendant’s rights were violated by the homicide instructions. The Due Process Clause requires in all criminal cases that the state prove guilt beyond a reasonable doubt. The prosecution must bear the burden of proving all of the elements of the crime charged. Thus, the Supreme Court has held that if “malice aforethought” is an element of murder, the state may not require the defendant to prove that he committed the homicide in the heat of passion, because heat of passion negates malice and in effect requires the defendant to disprove the element of malice aforethought. [Mullaney v. Wilbur (1975)] Here, the judge’s instructions create the same result with “extreme emotional disturbance.” By instructing the jury that it may presume the existence of malice aforethought from proof of an unlawful killing, and that the defendant bears the burden of showing that he did not act with malice, the judge improperly imposed on this particular defendant the burden to disprove an element of murder. Thus, (B) and (D) are incorrect. Insanity is an affirmative defense for which it is constitutional to impose the burden of proof on the defendant. Although many jurisdictions require a defendant to prove his insanity by a preponderance of the evidence, federal courts require proof by clear and convincing evidence, and one Supreme Court case upheld requiring a defendant to prove insanity beyond a reasonable doubt. [See Leland v. Oregon (1952)] Thus, (B) and (C) are incorrect and (A) is correct.

74
Q

A defendant who was indigent was charged with a crime for which the maximum punishment is six months in prison and a fine of $500. At a hearing, the defendant told the judge that he wanted to plead not guilty and that he wished to represent himself. The judge told the defendant that the court would appoint an attorney to represent him if he needed counsel. He further explained that the state has a policy of appointing private attorneys to defend indigents, and that if such defendants are acquitted or imprisoned, there is no charge for the court-appointed lawyer. If, however, the defendant is sentenced to probation, the defendant must pay “reasonable attorneys’ fees,” which it is presumed the defendant will be able to pay out of job earnings while on probation. The defendant told the judge that paying for an attorney would be difficult, and that he still wished to defend himself. The judge believed that the defendant was competent to defend himself, but nevertheless appointed an attorney with criminal defense experience to defend him. The defendant fully cooperated with the attorney, who did a highly competent job, but the evidence heavily favored the state. The defendant was convicted, but the attorney’s plea for leniency was effective, and the defendant received a suspended sentence and probation.

Two weeks later, the defendant received a bill for $500 for legal services, a figure that represented about half the sum a lawyer not appointed by the court would have charged for similar work. Although the defendant would be able to pay the bill over a long period of time via installments, he was angry that he had been billed at all, and believed that he could have gotten probation if he had been allowed to argue his own case. He consulted another attorney, and asked her to appeal both his conviction and the imposition of the legal fees.

If the second attorney files an appeal, what is the appellate court most likely to do?

A
Affirm both his conviction and the imposition of fees, because there is no reversible error, since the trial attorney competently represented the defendant and the state has a right to recoup costs from those able to pay.

B
Affirm the conviction because there was no reversible error, but reverse the imposition of fees, because the defendant could have gotten probation for himself as easily as the attorney did.

C
Reverse the conviction, because the defendant was denied the right to defend himself, but affirm the imposition of fees because the attorney was competent and the state has a right to recoup costs from those able to pay.

D
Reverse both the conviction and the imposition of fees, because the defendant was denied the right to defend himself.

A

CORRECT ANSWER: D. Reverse both the conviction and the imposition of fees, because the defendant was denied the right to defend himself.

D is Correct. The appellate court would likely reverse both the conviction and the imposition of fees. Although a waiver of the right to counsel will be carefully scrutinized to ensure that the defendant has a rational and factual understanding of the proceeding against him, a defendant has a right to waive counsel and represent himself as long as the waiver is knowing and voluntary and he is competent to proceed pro se. Where the state provides counsel in cases of indigence, it may then seek reimbursement from a convicted defendant who subsequently becomes able to pay. Here, the judge thought that the defendant was competent to represent himself. Therefore, his waiver of counsel should have been honored. Violation of the defendant’s right to represent himself will result in a reversal of his conviction. Although, as stated above, the state may recoup costs of appointed counsel from indigents who become able to pay, the state cannot recover from the defendant because the attorney was appointed against the defendant’s will and in violation of his right to represent himself. Had effect been given to the defendant’s right to represent himself, there would have been no attorneys’ fees to be assessed against him. (A) is incorrect because the denial of the defendant’s right to represent himself does constitute reversible error, regardless of the competence with which the attorney represented him. Also, as explained above, the state’s right to recoup costs from indigents who become able to pay does not extend to those defendants who have had appointed counsel imposed on them against their will. (B) is incorrect because it states that the conviction involved no reversible error, when denial of the defendant’s right to represent himself is reversible error. (B) also incorrectly states that the imposition of fees should be reversed because the defendant could have gotten probation for himself as easily as his attorney did. There is no way to know that the defendant could have gotten probation for himself as easily as the attorney did, and it is not really at issue whether the defendant could have done so. What is at issue is that the costs were imposed as part of an unconstitutional forced representation by appointed counsel. Thus, even if the attorney obtained probation for the defendant when the defendant would have been unable to do so, the costs should be reversed, because the defendant should have been free to proceed pro se, even if this meant risking imprisonment. Similarly, (C) is incorrect because it states that the attorney’s competence is a reason for affirming the imposition of fees. As stated above, the defendant had the right to proceed pro se if his decision was knowing and voluntary and he was competent to proceed pro se, regardless of the fact that appointed counsel may have been highly competent. The state cannot force a defendant to forgo a constitutionally protected right by accepting appointed counsel, and then further force him to pay for such counsel.

75
Q

The police obtained a valid warrant to arrest a suspected drug smuggler at his home. Two officers went to the house, rang the doorbell, and knocked at the front door. They heard movement inside, but no one answered. The officers proceeded to enter and apprehended and arrested the suspect in the dining room as he was hurriedly placing clear plastic bags into a suitcase. While one of the officers seized the clear plastic bags, the second officer searched the rest of the house and found several bricks of marijuana on a desk in the man’s bedroom. The clear plastic bags were later determined to contain cocaine.

The defendant was charged with unlawful possession of drugs. He moved to suppress both the cocaine and the marijuana evidence based on unlawful police conduct.

How should the court rule on the defendant’s motion?

A
Grant the motion with respect to the marijuana but deny it as to the cocaine.

B
Grant the motion with respect to the cocaine but deny it as to the marijuana.

C
Grant the motion with respect to both the marijuana and the cocaine.

D
Deny the motion with respect to both the marijuana and the cocaine.

A

CORRECT ANSWER: A. Grant the motion with respect to the marijuana but deny it as to the cocaine.

(A) is correct. The court should grant the motion with respect to the marijuana but deny it as to the cocaine. When police officers have a valid arrest warrant, they may enter a home—even if the occupants do not voluntarily admit the officers—to execute the warrant. Once inside, under the plain view exception to the search warrant requirement, the police may seize any contraband or evidence in plain view of a place they have a right to be to execute the warrant. Here, the bags of cocaine were apparently in the dining room and in plain view when the police entered the dining room to arrest the defendant. However, once the defendant was arrested, the police officers had no right to proceed to search the home. Incident to the arrest, the officers could search the defendant’s wingspan (the area he could reach into as the police took him out of the house). They could also make a limited sweep to look for others who might be hiding and present a danger to the officers if the officers had a reason to believe such others might be present. But here, nothing in the facts indicates that the bedroom in which the marijuana was found was on the way out of the house or that the officers had reason to believe any dangerous person was in the house. Therefore, the marijuana was seized in an unconstitutional search and must be suppressed. For this reason, (B), (C), and (D) are all incorrect.

76
Q

A farmer was arrested after selling her surplus fruits and vegetables for several days at a vacant lot in the nearby town. A statute provides that it is a misdemeanor, punishable by a fine of up to $500 and/or imprisonment in county jail for up to one year, to sell any product without a business license, except for informal sales held on the property of the seller no more often than once every three months. At the farmer’s trial, she requested but was refused appointed counsel.

Assuming that she would otherwise qualify as indigent, if she is convicted of violating the statute, what is the maximum penalty that may be imposed on her?

A
Imprisonment for six months.

B
A $500 fine.

C
Imprisonment for six months and a $500 fine.

D
No penalty, because her conviction is void as having been obtained in violation of her right to counsel under the Sixth Amendment.

A

CORRECT ANSWER: B. A fin of $500.

B is Correct. The farmer may be fined under the statute but not imprisoned. The right to counsel under the Sixth Amendment gives the defendant the right to be represented by privately retained counsel or to have counsel appointed for her by the state if she is indigent. However, the right to counsel applies to misdemeanor trials only when a sentence of imprisonment is actually imposed (including a suspended sentence). Thus, even though the misdemeanor statute permits a potential jail term, its alternative penalty of a fine may constitutionally be imposed on the defendant despite the refusal to provide her with counsel. (A) and (C) are incorrect because the right to counsel would apply to any misdemeanor trial in which imprisonment is actually imposed. The failure to provide the farmer with counsel would preclude the imprisonment sentence. (D) is incorrect because, as discussed above, the misdemeanor’s alternative penalty of a fine may be imposed without violating the constitutional right to counsel at trial.

77
Q

With the owner’s permission, a defendant took the owner’s car with the intention of driving it three miles to a grocery store and back. While on his way to the store, the defendant decided to keep the car. Two days later, he changed his mind and returned the car. The defendant is charged with larceny of the car.

Should the defendant be found guilty of larceny?

A
Yes, because he kept the car for two days.

B
Yes, because he had the requisite intent.

C
No, because he returned the car two days later.

D
No, because he lacked the requisite intent.

A

CORRECT ANSWER: D. No, because he lacked the requisite intent.

The defendant is not guilty of larceny because he lacked the intent to deprive the owner permanently of his car at the time of the taking. Larceny consists of: (i) a taking (ii) and carrying away (iii) of tangible personal property (iv) of another (v) by trespass (vi) with intent to permanently (or for an unreasonable time) deprive the person of his interest in the property. The intent to deprive must exist at the time the property is taken. Here, the defendant is not guilty of larceny because, at the time of the taking, he intended to return the car within a reasonable time and had a substantial ability to do so. Note also that the continuing trespass doctrine does not apply here. Under that doctrine, if a defendant takes property with a wrongful state of mind but without the intent to steal, and later, while still in possession of it, forms the intent to steal it, the trespass involved in the initial wrongful taking is regarded as “continuing” and the defendant is guilty of larceny. Here, the defendant did not take the owner’s car with a wrongful state of mind; he intended to return it. Thus, (A) and (B) are incorrect. (C) is incorrect because if the defendant had the requisite intent at the time of the taking, he would be guilty of larceny even though he later returned the car.

78
Q

To raise revenue, a city erected billboards on the sides of all government buildings and planned to sell the space for commercial advertising. A city ordinance provided that any advertiser could rent the space, provided the activity or product advertised was legal and had “nothing to do with politics” because the city sought to “avoid controversy.”

The owner of a bookstore that specialized in political books sought to lease a billboard on a city building to place an ad. In addition to selling books, the owner conducted daily reading and study groups in the store on various political philosophies. The proposed ad implored onlookers to come to the bookstore to study communism at the store at 6 p.m. nightly. The ad was rejected by city officials.

If the owner files an appropriate suit against the city in federal district court asserting violation of her First Amendment rights, is she likely to prevail?

A
Yes, because the city has made the sides of civic buildings public forums.

B
Yes, because restrictions on commercial speech must be narrowly tailored to serve a substantial government interest directly advanced by the restriction.

C
No, because the restriction is viewpoint neutral and reasonably related to a legitimate government purpose.

D
No, because the restriction is content neutral and narrowly tailored to serve an important government interest.

A

CORRECT ANSWER: C. No, because the restriction is viewpoint neutral and reasonably related to a legitimate government purpose.

The restriction will probably be upheld because it is viewpoint neutral and reasonably related to a legitimate government purpose. The billboards are not traditional public forums; rather, they will be found to be “commercial ventures” by the city. In a similar setting, the Supreme Court held that cities could differentiate between broad categories of speech in accepting advertising on city-owned buses (i.e., the Court allowed a city to refuse political advertising and accept only commercial advertising) [see Lehman v. Shaker Heights (1974)], as long as the restriction was viewpoint neutral and reasonably related to a legitimate government interest. The city rule here is constitutional because it is viewpoint neutral (it distinguishes between broad categories of speech but does not distinguish based on content within a category) and it is reasonably related to the legitimate government interest of avoiding controversy. (A) is incorrect because the Supreme Court has held that allowing advertising on government-owned property does not make that property a public forum; rather, the property is a commercial forum. [See Lehman v. Shaker Heights, supra] (B) is incorrect because, although it states the general rule for regulation of commercial speech, the more specific Supreme Court precedents regarding advertising on city-owned property used for a proprietary venture (discussed above) would apply here. (D) is incorrect because it states the rule for traditional public forums, which this is not. Also, this restriction is not content neutral. It excludes political speech, which is a content-based (but not viewpoint-based) distinction.

79
Q

A housing development contained one-, two-, and three-bedroom units. All units were suitable for occupancy, and the developers of the project filed the appropriate documents, including a Declaration of Restrictions that limited ownership and occupancy of the units to families or to groups of unrelated adults of not more than three in number. Each deed to the individual units also contained the restriction.

One of the two-bedroom units was purchased by a woman and her boyfriend. They immediately moved into the unit with another unmarried couple who were friends of theirs. Other unit owners brought suit against the woman and her boyfriend to enjoin the occupancy by the other couple.

If the other unit owners prevail, what will likely be the reason?

A
The litigants are private parties and the restriction was not enacted by the government.

B
Enforcement of the restriction is rationally related to a legitimate government interest.

C
Notice was not given by the woman and her boyfriend to the sellers of the unit that they intended to occupy the residence with another couple.

D
The restriction constitutes a lawful restraint on alienation.

A

CORRECT ASNWER: B. Enforcement of the restriction is rationally related to a legitimate government interest.

If the other owners prevail, it will be because the court’s enforcement of the regulation is rationally related to a legitimate government interest. The Supreme Court has held that court enforcement of restrictive covenants in deeds constitutes state action, and thus a court may enforce a restrictive covenant only if it is constitutional. Under the Due Process Clause, unless fundamental rights are involved, government action is constitutional as long as it is rationally related to any conceivable legitimate end of government. Zoning regulations prohibiting three or more unrelated persons from living together have been held not to infringe on any fundamental rights in Village of Belle Terre v. Boraas (1974); hence, judicial enforcement of a private covenant with that prohibition likely would also be valid for that reason. (A) is wrong because, as discussed above, government action includes court enforcement of restrictive covenants between private parties, and deed restrictions based on race have been held unenforceable in Shelley v. Kraemer (1948), because enforcement would constitute government action supporting discrimination against a suspect class. Here, however, no suspect class or fundamental right is involved. (C) is wrong because it is not relevant to a determination of whether the restriction can be properly enforced by the court. (D) is wrong because the issue here involves occupancy of the unit rather than transfer or sale.

80
Q

A state enacted a statute requiring the parents of every child to have the child vaccinated for smallpox before the child’s third birthday. Failure to comply was a misdemeanor punishable by a fine of $500 or six months in the county jail. The parents of a nine-month-old infant living within the state objected to application of the statute to their child on the ground that any injections or vaccinations violated the tenets and beliefs of their religion. The parents have commenced an action in federal court to declare the statute unconstitutional on the ground that it violates their right to the free exercise of their religion under the First Amendment to the United States Constitution.

What is the best argument to defeat their action?

A
A proceeding for declaratory judgment is not the proper vehicle for asserting this claim.

B
The state legislature has repeatedly defeated bills to repeal the statute.

C
There is no substantial threat that the statute will be enforced.

D
The federal courts should abstain until the state courts have had an opportunity to construe the statute.

A

CORRECT ANSWER: C. There is no substantial threat that the statute will be enforced.

C is Correct. If there is no substantial threat that the statute will be enforced, then there are no constitutional issues ripe for review. A federal court will resolve only constitutional issues that are necessarily presented, ripe for review, and unavoidable for decision of the case. Someone seeking a declaration that a statute is unconstitutional must demonstrate that she has engaged (or will engage) in specific conduct, and that the challenged statute poses a real and immediate danger to her interests. The court will not determine the constitutionality of a statute when the statute has not been enforced and there is no immediate threat that it will be enforced. Thus, if the state statute is not likely to be enforced, it is not a substantial threat to the parents and so they will be unable to demonstrate any real and immediate harm (or threat thereof) to their interest. Here, the parents’ child is nine months old and the statute cannot be violated before the child turns age three. Therefore, this case is not ripe, and this choice gives the state a chance to defeat the parents’ argument. The other choices do not present the state with viable arguments. (A) is incorrect because, if ripeness were present, an action for declaratory judgment would be proper. A federal court may not issue advisory opinions, but where there is a real controversy, it may issue a final judgment declaring the rights and liabilities of the parties even though no affirmative relief is sought. Maintenance of such an action requires an actual dispute between parties with adverse legal interests. If the parents were being prosecuted for violating the statute (or threatened with prosecution), the requisites for a declaratory judgment would be met and it would be an appropriate means of determining the constitutionality of the state statute. (B) is incorrect because the refusal of the state legislature to repeal the statute indicates a legislative intent that the statute remain in effect, thus implying a greater likelihood that the statute would be enforced. This would make the parents’ case ripe and so would help them rather than defeat their action. (D) is incorrect because the grounds for abstention are absent. When a federal constitutional claim is premised on an unsettled question of state law, the federal court should abstain, to give the state courts a chance to settle the underlying state law question. Here, there is no unsettled question of state law. Therefore, abstention by the federal court would be inappropriate.

81
Q

A state statute prohibited individuals from donating more than $1,000 per year to any group that lobbies for or against any matter up for consideration before the state legislature. A voter who wanted to donate $5,000 to a lobbying group challenged the statute on constitutional grounds in federal court.

Is the court likely to uphold the statute?

A
Yes, because the statute is reasonably related to the state’s legitimate interest in controlling such contributions.

B
Yes, because the statute does not restrict the core political speech right to donate directly to legislative candidates.

C
No, because the statute places a restraint on core political speech and association rights without sufficient justification.

D
No, because a state may not place any limits on the amount of money that may be contributed to political campaigns.

A

CORRECT ANSWER: C. No, because the statute places a restraint on core political speech and association rights without sufficient justification.

C is Correct. The federal court would likely find that the statute violates the First Amendment by restraining political speech and association rights. While the government may limit the amount of contributions that an individual may contribute to a candidate’s campaign, it may not limit contributions to groups that lobby for or against matters before the legislature, because the Supreme Court has found that such a law does not serve a sufficiently important interest to outweigh the restraints that it puts on the First Amendment freedoms of speech and association. (A) is incorrect because it states an incorrect result (the statute likely will be found invalid for the reason stated above) and because it states an improper standard (a statute limiting campaign contributions is tested under intermediate scrutiny—it must be closely drawn to achieve a sufficiently important interest). (B) is incorrect. As mentioned above, the government may limit the amount of contributions that an individual may contribute to a candidate’s campaign, because the government has a sufficiently important interest in preventing the corruption or the appearance of corruption from large contributions that outweighs First Amendment issues. (D) is incorrect because it is too broad. While states may not limit the amount that a person may spend to get a candidate elected, they may limit the amount that a person may contribute to another’s campaign.

82
Q

An attorney was employed by the United States Department of Health and Human Services in a regional office located in a tobacco-growing state. A labor contract between the agency and the clerical workers union contained a policy providing for termination of union employees only for certain specified grounds. The attorney, however, was not a member of the union and not covered by such a policy. The attorney was angered by the regional director’s refusal to adopt a no-smoking policy for employees and visitors in the office. She posted a notice in the employee cafeteria ridiculing what she called the hypocrisy of an agency promoting health issues and nonsmoking programs while refusing to provide its employees with those same opportunities. The notice prompted a great deal of debate among the employees and was brought to the attention of the regional director, who was very displeased.

Which of the following statements is most accurate regarding the director’s right to dismiss the attorney?

A
The attorney has a liberty interest in the exercise of her First Amendment rights that entitles her to a hearing to contest the grounds of her dismissal.

B
The attorney has a property interest as a public employee that precludes her from being fired without notice and an opportunity to respond.

C
The attorney has no right to a hearing because her statements were not an expression of views on public issues.

D
The attorney has both a liberty interest and a property interest that entitles her to a pre-termination evidentiary hearing.

A

CORRECT ANSWER: A. The attorney has a liberty interest in the exercise of her First Amendment rights that entitles her to a hearing to contest the grounds of her dismissal.

If the attorney is fired, she has a right to a hearing to determine whether her First Amendment rights were violated by her dismissal. Under the Due Process Clause of the Fifth Amendment, a person has a liberty interest in the exercise of specific rights provided by the Constitution, including freedom of speech. If a government employer seeks to fire an employee for speech-related conduct when the speech involved a matter of public concern but is not made pursuant to her official duties, the courts must carefully balance the employee’s rights as a citizen to comment on a matter of public concern against the government’s interest as an employer in the efficient performance of public service. Under the Court’s expansive interpretation of what is a public issue in this context [see Rankin v. McPherson (1987)], the attorney’s statement would probably qualify. At the very least, she can make a sufficient showing that her termination violates her free speech rights to be entitled to a hearing on the issue under procedural due process principles. [See Givhan v. Western Line Consolidated School District (1979)] (B) is wrong because the attorney does not appear to have a property interest in her job. A public employee who is subject to removal only for “cause” has a property interest in her job and generally must be given notice of the charges against her that are to be the basis for her job termination, and a pre-termination opportunity to respond to those charges. Here, however, the attorney did not have a property interest in her job; she could have been dismissed for no reason at all. She was not covered by the labor contract between the agency and its clerical workers, and there appears to be no other basis for her to claim an entitlement to continued employment. (C) is wrong because the attorney is entitled to a hearing as long as she can raise a prima facie claim that her speech, which was regarding an important health issue and the perception of her agency, was on a public issue and therefore protected by the First Amendment. (D) is wrong for two reasons. As discussed above, the attorney does not have a property interest in her job. Also, due process does not necessarily entitle her to a pre-termination evidentiary hearing; a post-termination evidentiary hearing is probably sufficient. [See Cleveland Board of Education v. Loudermill (1985)]

83
Q

In response to lobbying, Congress passed legislation appropriating $200 million for grants-in-aid to domestic horse liniment manufacturers and providing some degree of protection from foreign competition. However, because of concern about inefficiencies in the industry, the legislation was amended to allow the Secretary of Commerce the authority to deny grants to horse liniment manufacturers who failed to meet certain “management efficiency standards” outlined in the legislation.

One liniment manufacturer and a member of the trade association petitioned the Secretary of Commerce for a $15 million grant. This figure equaled the amount it would be entitled to under the legislation, based on the number of its employees, plants, and upon its average production of horse liniment over a 10-year period. The Secretary of Commerce refused to award the funds, because she determined that the liniment manufacturer was making no attempt to improve its management efficiency. The liniment manufacturer filed suit against the Secretary of Commerce, asserting that the power granted to the Secretary was unconstitutional.

Is the legislation constitutional?

A
Yes, because the Secretary of Commerce, as a representative of the executive branch, may be granted regulatory authority.

B
Yes, because the executive branch, represented by the Secretary of Commerce, shares power with Congress in the field of foreign commerce.

C
No, because there was an improper delegation of legislative power.

D
No, because the executive branch may not impound funds appropriated by Congress.

A

CORRECT ANSWER: A. Yes, because the Secretary of Commerce, as a representative of the executive branch, may be granted regulatory authority.

A is Correct. Congress may delegate many of its powers to executive agencies, provided adequate standards are established to govern exercise of the delegated power. The power was properly delegated to the Secretary of Commerce because the statute “outlines” the “management efficiency standards” to be followed. (B) is incorrect because Congress has sole authority over foreign commerce; the authority is not shared with the executive branch. (C) is incorrect because the delegation is proper if adequate standards are established; as determined above, there are adequate standards here. (D) is incorrect because there is no executive impoundment here, merely a refusal to grant funds based upon standards established by Congress. Such standards are proper where, as here, Congress acts pursuant to its spending power and the standards are specified in advance.

84
Q

To reduce incidents of violence among male gang members at a youth center, a city passed an ordinance forbidding any male between the ages of 13 and 19 to enter the center unless accompanied by a female.

An 18-year-old male who was refused admission at the center because he was not escorted by a female filed suit in federal court to strike down the ordinance as unconstitutional.

Should the court find the city ordinance constitutional?

A
Yes, because in these situations the government is performing a parens patriae function.

B
Yes, if the city can show a rational relationship between the ordinance and maintaining order at the center.

C
No, if the plaintiff can show that the ordinance was not necessary to promote a compelling government interest.

D
No, unless the city can show that the ordinance is substantially related to important government objectives.

A

CORRECT ANSWER: D. No, unless the city can show that the ordinance is substantially related to important government objectives.

The court should find the ordinance unconstitutional unless the city shows that the ordinance is substantially related to important government objectives. Classifications based on gender, such as the ordinance here, are quasi-suspect and violate equal protection unless they are substantially related to an important government objective. (A) is incorrect; even if the government were performing a parens patriae function, it would not be permitted to violate equal protection. (B) states the wrong equal protection test; because a quasi-suspect class is involved, the rational relationship test does not apply. (C) states the wrong test and places the burden of proof on the wrong party.

85
Q

The United States Surgeon General was cited for contempt for refusing to answer questions as part of a Senate investigation regarding an issue in the Food and Drug Administration.

His contempt citation will be dismissed if he can show which of the following?

A
As a member of the executive branch, he is immune from prosecution.

B
If he answered the questions, he could be subject to dismissal from his position as Surgeon General.

C
The questions do not relate to any matter concerning which the Senate may legislate.

D
The questions do not relate to any matter concerning current or planned legislation.

A

CORRECT ANSWER: C. The questions do not relate to any matter concerning which the Senate may legislate.

His contempt citation will be dismissed if he can show that the questions do not relate to any matter concerning which the Senate may legislate. Congress’s power to investigate is limited to matters on which it can legislate. Therefore, if the Surgeon General can demonstrate that the questions concerned matters upon which Congress could not legislate (not an enumerated power under Article I, Section 8), then this contempt citation must be dismissed. (A) is wrong because Congress can question a member of the executive branch concerning his duties, and he is not immune from prosecution. (B) is wrong because he would have a privilege not to answer only if he is subject to criminal liability. Merely because he may get fired is not sufficient grounds for him to refuse to answer a lawful question posed by a member of the Senate in an appropriate hearing. (D) is wrong because it is too narrow. As explained above, Congress may investigate any matter on which it may legislate; current or planned legislation is not required.

86
Q

A landowner was the owner in fee simple of a tract of land. The landowner conveyed the tract to her neighbor, “for life and then to the first child of my neighbor’s only sister who shall reach the age of 21.” The sister was unmarried and childless at the time of the conveyance. Five years later, the sister married, and she gave birth to a son the following year. When the son reached the age of 21, he brought an action in ejectment against the neighbor, who is still alive and living on the tract of land.

Which of these is the neighbor’s best defense against the son’s attempt to eject her?

A The neighbor’s life estate was not subject to termination during her lifetime.

B The son has no claim to the tract, because he was not living at the time of the landowner’s original conveyance.

C A conveyance of the tract to the son violates the Rule Against Perpetuities.

D The son’s interest, if any, would be that of a contingent remainder, and the contingency is the neighbor’s death.

A

CORRECT ANSWER: A. The neighbor’s life estate was not subject to termination during her lifetime.

The neighbor’s best defense against the son’s attempt to eject her is that her life estate was not subject to termination during her lifetime. The usual life estate is measured by the life of the grantee. Although a life estate may be made defeasible (e.g., determinable, subject to a condition subsequent, or subject to an executory interest), the conveyance here created an indefeasible life estate (i.e., one that will end only when the life tenant dies) in the neighbor. (B) is incorrect because the son did not have to be alive at the time of the original conveyance in order to now claim an interest in the tract. (C) is incorrect because the son’s interest would vest within 21 years of a life in being (his mother, the sister). (D) is incorrect because the son’s interest would be a vested rather than contingent remainder. A remainder is contingent if it is created in favor of unborn or unascertained persons. Because the sister was childless at the time of the conveyance, the remainder to the sister’s “first child . . . who shall reach the age of 21” was a contingent remainder. It remained contingent at the son’s birth because he had to reach age 21 to take. However, the son has reached age 21, and thus his remainder is vested. It is indefeasibly vested because it is not subject to being defeated, divested, or diminished in size.

87
Q

A fee simple owner of a restaurant provided in his will that the property should go on his death “in fee simple to my friend, but if during my friend’s lifetime my son has children and those children are alive when my friend dies, then to said living children.” When the owner died, the friend took over the restaurant.

If the son has children and one or more of them are alive when the friend dies, who will take title to the restaurant at that time?

A The friend’s heirs, because the attempted gift to the son’s children is invalid under the Rule Against Perpetuities.

B The son’s children, because their interest is not contingent, being a possibility of reverter.

C The son’s children, because their interest is vested, subject to defeasance.

D The son’s children, because their interest will vest, if at all, within a life in being plus 21 years.

A

CORRECT ANSWRE: D. D The son’s children, because their interest will vest, if at all, within a life in being plus 21 years.

The interest given to the son’s children does not violate the Rule Against Perpetuities because the interest will vest, if at all, within 21 years after the life of the friend. Pursuant to the Rule Against Perpetuities, no interest in property is valid unless it must vest, if at all, not later than 21 years after one or more lives in being at the creation of the interest. In the case of a will, the perpetuities period begins to run on the date of the testator’s death, and measuring lives used to show the validity of an interest must be in existence at that time. Here, the interest given to any of the son’s children who are born during the friend’s lifetime and who survive the friend must vest, if at all, on the death of the friend (who is a life in being at the time of the owner’s death). Thus, this interest will vest, if it does vest, within 21 years after the friend’s life, and is therefore not in violation of the Rule Against Perpetuities. (A) is therefore incorrect; if one or more of the son’s children is alive at the time of the friend’s death, the friend’s heirs will get nothing because their fee simple will be divested. (B) incorrectly characterizes the interest of the son’s children as a possibility of reverter. A possibility of reverter is the future interest left in a grantor who conveys a fee simple determinable estate. Although under different circumstances the son’s children could acquire a possibility of reverter as heirs of the grantor (the owner), their interest in this case was conveyed directly to them in the owner’s will. (C) is incorrect because the interest of the son’s children is not vested. Their interest is a shifting executory interest rather than a remainder because it divests the fee simple estate of the friend and his heirs. The friend has a fee simple subject to an executory interest because the estate will remain with his heirs if none of the son’s children are alive when the friend dies. The friend’s death while the son’s children are alive divests the interest of the friend’s heirs; it is therefore a shifting executory interest rather than a remainder.

88
Q

A husband was on his way to meet his wife for lunch at the restaurant in the lobby of a bank building where she worked. He entered the building, which was owned and operated by the bank, and started walking towards his wife, who he could see waiting for him at a table near the rear of the restaurant. Suddenly he heard the sound of breaking glass from the restaurant area and saw a large piece of artwork made of stained glass fall onto the seating area of the restaurant. Immediately thereafter he saw several injured persons, including his wife, lying in the wreckage of the artwork. He fainted and hit his head on the marble floor, fracturing his skull. The artwork had collapsed because the pedestal that the bank had provided for the artwork was not properly constructed.

If the husband sues the bank for his injury, is he likely to prevail?

A No, because he was not personally in the zone of danger of physical injury.

B No, because his skull fracture was not foreseeable.

C Yes, because his wife was one of the persons he saw lying in the wreckage.

D Yes, because the bank had provided the pedestal for the artwork.

A

CORRECT ANSWER: C. Yes, because his wife was one of the persons he saw lying in the wreckage.

The husband will recover for his injuries because his wife was among those injured by the collapse of the artwork. The duty to avoid negligent infliction of emotional distress may be breached when the defendant creates a foreseeable risk of physical injury to the plaintiff. In most jurisdictions, a bystander who sees the defendant negligently injuring another can recover for his own distress if (i) the plaintiff and the person injured by the defendant’s negligence are closely related, (ii) the plaintiff was present at the scene of the injury, and (iii) the plaintiff personally observed or perceived the event. Observation is typically by sight, but may also be by hearing or other senses under certain circumstances. Here, the husband heard the screams and the sound of breaking glass when the artwork collapsed as he entered the lobby. Even though he evidently did not see the artwork collapse on the diners, he heard it crash where his wife was sitting and saw the immediate aftermath. Because his wife was one of the persons injured by the collapse of the artwork, he can recover damages for the injuries caused by his distress. (A) is incorrect because, as stated above, the majority rule allows a bystander to recover based on the factors stated above even if he is outside the zone of danger of physical injury. (B) is incorrect because injury to the husband was foreseeable. The husband is a foreseeable plaintiff in a claim for negligent infliction of emotional distress because of the injury to his wife, as discussed above. The unusual manner in which his physical injury occurred is irrelevant to the defendant’s liability. (D) is incorrect because it does not matter that the bank had provided the pedestal. Even if the negligent construction of the pedestal had been done by a third party, the bank remains liable to invitees on its premises because a business has a nondelegable duty to keep its premises safe for customers.

89
Q

Three drivers were in an automobile accident in a city in State A. The drivers were citizens of State A, State B, and State C. The State B driver filed a tort action against the other two in a State A state court, seeking $300,000 for her severe injuries. The State C driver wants to remove the action to a federal district court.

Is the action removable?

A. No, because one of the defendants is a citizen of State A.

B. No, because an action may be removed from state court only if it “arises under” federal law.

C. Yes, because there is complete diversity of citizenship and the amount in controversy exceeds $75,000.

D. Yes, because one of the defendants is a citizen of a state other than State A.

A

CORRECT ANSWER: A.

The action is not removable. Under 28 U.S.C. section 1441, a defendant may remove an action that could have originally been brought in the federal courts. (In other words, subject matter jurisdiction based on either a federal question being presented or on diversity of citizenship would have been present had the case been filed in federal court.) However, a case may not be removed on the basis of diversity jurisdiction if a defendant is a citizen of the state in which the action was filed. Here, the action was filed in State A against a State A defendant; thus, the case may not be removed, even though it is a State C defendant who is seeking removal. This fact also makes (D) incorrect. (B) is incorrect because a case may be removed based on diversity, with the restriction that removal is not available if one of the defendants is a citizen of the forum state. (C) is incorrect. Even though the case satisfies the requirements of diversity jurisdiction, the “in-state defendant” restriction prevents removal.

90
Q

The owner of a boat took two friends out on a lake near his home. One of his friends was driving the boat when it struck a partially submerged rock that the owner of the boat had forgotten to tell him about. The owner of the boat and the other passenger were injured; the driver of the boat was not hurt.

In a jurisdiction that applies joint and several liability with comparative contribution, the passenger brought suit against both the boat owner and the driver, and the boat owner also sued the driver. The jury determined that the boat owner was 55% at fault and suffered $10,000 in damages, the driver of the boat was 45% at fault, and the injured passenger suffered $100,000 in damages. After entry of judgment, the boat owner paid the passenger her total damages of $100,000, while the driver of the boat has paid nothing.

How much, if anything, can the boat owner recover from the driver?

A. $45,000, because the driver was 45% at fault.

B. $49,500, because the driver was 45% at fault and the boat owner suffered $10,000 in damages.

C. $50,000, because the boat owner and the driver are jointly liable.

D. Nothing, because the boat owner was more at fault than the driver.

A

CORRECT ANSWER: B. $49,500, because the driver was 45% at fault and the boat owner suffered $10,000 in damages.

The boat owner can recover $45,000 through comparative contribution for the passenger’s claim and $4,500 on his own claim against the driver of the boat. Most comparative negligence states have adopted a comparative contribution system based on the relative fault of the various tortfeasors. Nonpaying tortfeasors who are jointly and severally liable are required to contribute only in proportion to their relative fault. Here, because the jurisdiction retained joint and several liability, the boat owner had to pay the passenger all of her damages. Under comparative contribution rules, the boat owner can obtain contribution from the driver for 45% of that amount, because the driver was 45% at fault. In addition, the boat owner has a direct claim against the driver for his own damages of $10,000, reduced by 55%, the amount of his fault. Thus, the total amount that the boat owner can recover from the driver is $49,500, making (B) correct and (A) incorrect. (C) is incorrect because it reflects traditional contribution rules, in which all tortfeasors were required to pay equal shares regardless of their respective degrees of fault. (D) is incorrect because a tortfeasor who was jointly and severally liable is not precluded from recovering contribution merely because he was more at fault than the other tortfeasors.

91
Q

An attorney came to work on a Saturday. When he signed in, he was advised by the morning security guard employed by the building management that he must be out of the building by 5 p.m., when it closes. However, he stayed past 5 p.m. to complete a brief that had to be filed on Monday morning. At 5:15 p.m., the afternoon security guard set the locks on all the doors of the building and left. Because she was in a hurry, she did not check the sign-in sheet to make sure that everyone had signed out, contrary to mandatory procedures. When the attorney tried to exit 15 minutes later, he discovered that the doors were all locked and could not be opened from the inside. He used his cell phone to call for help, and a supervisor from the building arrived and let him out shortly thereafter.

If the attorney sues the building management for false imprisonment, is he likely to win?

A. Yes, because the guard acted recklessly by locking the doors and leaving without checking that everyone was out of the building.

B. No, because the attorney became a trespasser by staying in the building past 5 p.m.

C. No, because the guard did not know that the attorney was locked in the building.

D. No, because the attorney suffered no harm from the confinement.

A

CORRECT ANSWER: C. No, because the guard did not know that the attorney was locked in the building.

The attorney will lose because the guard did not know that he was still in the building. For false imprisonment, the plaintiff must show (i) an act or omission on the part of the defendant that confines or restrains the plaintiff to a bounded area, (ii) intent on the part of the defendant to confine or restrain the plaintiff, and (iii) causation. Here, because the guard apparently did not know that the attorney was still in the building, she had no intent to confine him when she locked the doors. (A) is incorrect because recklessness is not enough; while the attorney likely has a cause of action for negligence against the guard, and through respondeat superior, the building, his claim is for false imprisonment. For liability for false imprisonment, there must be an intent to confine. (B) is incorrect because his status as a trespasser, while it may otherwise make him liable to the building for trespass, does not preclude him from recovering for false imprisonment. (D) is incorrect because the attorney need not show harm from the confinement to recover for false imprisonment, as long as he was aware of the confinement.

92
Q

A tenant vacated an apartment because he could no longer afford the rent. To ensure that the delinquent tenant made up for past arrearages, the landlord would not let him remove his personal property from the apartment. The tenant found a temporary place to stay with a friend, who wanted to help the tenant get his property back. The tenant remembered that the apartment would be vacant the upcoming weekend and that the landlord would be out of town, so he suggested that they break into the apartment and take the property then. They drove the tenant’s pickup to the apartment, and the friend entered through an unlocked window. The friend then opened the door for the tenant, and the pair collected the personal property. While the tenant was getting ready to drive away, the friend returned to the apartment and carried out some of the fixtures to the apartment. At this point, police officers who had been alerted by neighbors arrived and arrested the pair.

What is the tenant’s best defense to a charge of burglary?

A. There was no “entry,” because as an occupant of the apartment, he consented to the entry.

B. There was no breaking, because the window was unlocked.

C. There was no intent to commit a felony.

D. He only took his own property.

A

CORRECT ANSWER: C. There was no intent to commit a felony.

Absence of intent to commit a felony is the best defense. If the tenant intended merely to retrieve his property, he would have had no intent to commit a felony when he entered the apartment and thus could not be convicted of burglary. Common law burglary consists of: (i) a breaking; (ii) and entry; (iii) of the dwelling; (iv) of another; (v) at nighttime; (vi) with the intent of committing a felony therein. The tenant entered the apartment intending to retrieve his own property. Thus, the facts indicate that the only felony the tenant could have intended to commit at the time of entry would be larceny. Larceny consists of: (i) a taking; (ii) and carrying away; (iii) of tangible personal property; (iv) of another; (v) by trespass; (vi) with intent to permanently (or for an unreasonable time) deprive the person of his interest in the property. Larceny element (iv) would be missing here if the tenant intended to retrieve only his own property. Without the intent to commit a felony, no burglary exists. (A) is wrong because the tenant was no longer an occupant of the apartment and so could not consent to the entry. (B) is wrong because opening the closed but unlocked window was a breaking. (D) is not his best defense. The tenant could argue that the landlord did not have superior rights to the tenant’s property (despite a possible claim of arrearages) and that therefore he did not take the property of another, and also that he did not take part in his friend’s theft. However, he still would be guilty if he entered the apartment with the intent to commit a felony inside; the stronger defense of choice (C) specifically negates that intent.

93
Q

Ten years ago, a chemical manufacturer decided that it needed a safe place to store toxic waste that was a byproduct of its manufacturing processes. A highly reputable engineering company was hired and an exhaustive survey was performed to find a site. Once the site was selected, the chemicals were stored there underground, using state-of-the-art techniques. However, the surveyors failed to discover a crack in the rock of the storage area that was too small for their instruments to detect. Over a period of years the chemicals began to seep through the crack and into the water table below the surface. Tests disclosed that a nearby city’s water supply, all of which came from local wells that tapped into the water table, contained unacceptably high levels of chemical contaminants. The city was required to spend $5 million on a complex filtration system to remove the dangerous chemicals from its water.

The city brought an action against the chemical manufacturer to recover the cost of building the filtration system. At the end of the plaintiff’s case establishing the above facts, the manufacturer moved for a directed verdict.

Should the court grant the manufacturer’s motion?

A
Yes, because the city has not presented any evidence that the manufacturer breached a duty owed to the city.

B
Yes, if the manufacturer also established that the city’s water wells were drilled after the manufacturer stored the chemicals.

C
No, if the court determines as a matter of law that storage of the chemicals creates a foreseeable risk of serious harm even when reasonable care is exercised by all actors.

D
No, because the trier of fact could determine that storage of the chemicals creates a foreseeable risk of serious harm even when reasonable care is exercised by all actors

A

CORRECT ANSWER: C.

C is Correct. The court should deny the chemical manufacturer’s motion if it determines that its storage of chemicals was an abnormally dangerous activity, making the manufacturer strictly liable for any harm caused by the chemicals. The city can bring a private nuisance action against the chemical manufacturer for the substantial and unreasonable interference with the use of its water. Nuisances may be based on intent, negligence, or strict liability. A strict liability standard for engaging in an abnormally dangerous activity would apply where the activity (i) creates a foreseeable risk of serious harm even when reasonable care is exercised by all actors; and (ii) is not a matter of common usage in the community. Whether an activity is abnormally dangerous is a question of law that the court can decide on a motion for a directed verdict. Because the city has presented no evidence of negligence on the chemical manufacturer’s part, the only way that the city can survive the manufacturer’s directed verdict motion is if the court finds that a strict liability standard applies, as suggested by (C). (A) is incorrect. While no evidence of a breach of duty owed in negligence has been shown, the city has presented enough evidence to establish liability if the court finds that the manufacturer was engaged in an abnormally dangerous activity. The city has a cause of action because its use of the water table adjacent to the storage site makes it a foreseeable plaintiff. (B) is incorrect because the fact that the city drilled its wells after the manufacturer stored its chemicals would not bar the city from recovering and would have no effect on the manufacturer’s motion. This defense, called “coming to the nuisance,” is generally rejected by the courts. (D) is incorrect because, as discussed above, whether the elements making an activity abnormally dangerous are present is a question of law for the court rather than a question of fact.

94
Q

A motorcycle enthusiast who lived in a state wilderness area with rugged terrain purchased a motorcycle that was promoted as an all-terrain motorcycle in advertisements showing it going over very rugged terrain. However, the shock absorbers that were sold with the motorcycle as standard equipment were not designed for rough terrain and would not provide a safe ride under these conditions. The owner’s manual that came with the motorcycle stated that it should not be driven over rough terrain without equipping it with heavy-duty shock absorbers designed for that purpose. The next day, the purchaser took his motorcycle to the wilderness area and rode onto a trail. He crested a hill and landed hard, causing his shock absorbers to fail. The purchaser lost control and crashed, suffering serious and permanent injuries.

The purchaser brought a strict products liability action against the motorcycle’s manufacturer in a jurisdiction that does not apply its comparative negligence rules to these actions. At trial, he presented evidence of the advertisements and the fact that the shock absorbers installed on the motorcycle were dangerously inadequate under off-road conditions. The manufacturer presented evidence that the purchaser had received the owner’s manual with the warning about the shock absorbers and had disobeyed a posted state statute in the wilderness area forbidding motorized vehicles from leaving the roadway. At the close of the evidence, both parties move for a directed verdict.

What should the court do?

A
Deny both motions, because the jury could determine that the purchaser’s use of the motorcycle over rough terrain was foreseeable.

B
Grant the manufacturer’s motion, because the owner’s manual adequately warned of the unsuitability of the shock absorbers for off-road use.

C
Grant the manufacturer’s motion, because the purchaser was in violation of the law when he drove off of the road.

D
Grant the purchaser’s motion, because the shock absorbers were dangerously inadequate for the off-road conditions shown in the motorcycle’s advertisements.

A

CORRECT ANSWER: A.

The court should deny both motions because the jury should determine whether the purchaser’s misuse of the motorcycle was foreseeable. A strict products liability action requires plaintiff to establish that: (i) the defendant is a commercial supplier, (ii) the defendant produced or sold a product that was defective when it left the defendant’s control, (iii) the defective product was the actual and proximate cause of the plaintiff’s injury, and (iv) the plaintiff suffered damage to person or property. If the product was dangerous beyond the expectation of the ordinary consumer or a less dangerous alternative or modification was economically feasible, the supplier has supplied a defective product. Furthermore, while some products may be safe if used as intended, they may involve serious dangers if used in other ways. Courts require suppliers to anticipate reasonably foreseeable uses even if they are misuses of the product. In this case, the manufacturer has supplied its motorcycle with standard shock absorbers that are probably safe for use on the road. However, the advertisements promoted use of the motorcycle for off-road purposes, and purchasers may not have taken note of the warning in the owner’s manual. It is a question for the trier of fact to determine whether the manufacturer should have foreseen that purchasers would use the motorcycle on rough terrain without buying different shock absorbers. (B) is wrong because a simple warning of danger in an owner’s manual is not sufficient under the “feasible alternative” approach if it would not be effective to deter users of the motorcycle from using it on rough terrain. The jury will need to determine whether the manufacturer should have provided different shock absorbers or changed the way the motorcycle was advertised. (C) is wrong. The fact that the purchaser was in violation of the law when he drove off of the road may establish that his conduct was contributorily negligent. However, ordinary contributory negligence such as failing to discover a defect or guard against its existence is not a defense to a products liability action based on strict liability in jurisdictions that do not apply their comparative negligence rules to strict products liability actions. The type of contributory negligence where one voluntarily and unreasonably encounters a known risk, which is essentially assumption of risk, would be a defense to strict liability, but there is no indication that the purchaser learned of the risk (such as by reading the owner’s manual) and decided to take a chance anyway with the shock absorbers that he had. Hence, the purchaser’s driving off the road does not warrant granting the manufacturer’s motion. (D) is wrong because the fact that the shock absorbers were not safe for off-road use does not establish that they were so defective as to be unreasonably dangerous. The jury will need to determine whether the off-road use was sufficiently foreseeable to make the motorcycle unreasonably dangerous to users.

95
Q

A consumer purchased a portable electronic device from a retailer. The device was in the manufacturer’s original packaging when the consumer purchased it. Shortly thereafter, the consumer was using the device appropriately when it generated an electrical shock. The shock injured the consumer and caused the circuits of the device to overheat and melt, destroying the device.

The consumer brought an action based on strict liability against the manufacturer of the device. At trial, the consumer established the above facts. The consumer’s expert testified that the product was in a defective condition unreasonably dangerous to users. However, because the device was destroyed, the expert could not determine whether the accident was caused by a design defect or a manufacturing defect. The manufacturer’s expert testified that the device was not defective.

If the manufacturer moves for a directed verdict at the close of the evidence, how should the court rule?

A
Grant the motion, because the consumer did not show that the retailer was not at fault.

B
Grant the motion, because the consumer’s expert could not determine the exact nature of the defect.

C
Deny the motion, because the jury could decide that the harm was caused by a dangerous defect present in the device when it left the manufacturer.

D
Deny the motion, because the consumer was injured by the device.

A

CORRECT ANSWER: C.

C is Correct. The court should deny the motion. Strict liability requires proof only that an unreasonably dangerous defective condition in the defendant’s product caused the plaintiff’s injury. The consumer presented sufficient evidence that the product was dangerously defective and that the defect was the cause of her injury so that the case should go to the jury. If a defect is difficult to trace (such as when a product is destroyed), the plaintiff may rely on an inference that this type of product failure ordinarily would occur only as a result of a product defect. (A) is wrong both because the consumer does not need to show absence of fault by the retailer and because the retailer’s negligence would not cut off the manufacturer’s liability for supplying a defective product. (B) is wrong because, regardless of the absence of evidence on the exact nature of the defect, the jury could conclude, based on all of the facts, that the device was dangerously defective. (D) is wrong because showing that the consumer was injured by the device is not enough to establish liability. The jury must still determine that the device was in a defective condition unreasonably dangerous to users when it left the manufacturer, as stated by (C).

96
Q

A landowner operated a honey farm on her property adjacent to a busy state highway. The landowner had numerous hives for her honeybees that she carefully maintained and operated in compliance with all appropriate regulations. A motorist was driving his sports car on the highway at a high rate of speed after a rain shower when he lost control on the wet pavement. His car crossed in front of a motorcyclist who was going in the opposite direction, causing the motorcyclist to crash into a ditch on the side of the road. The motorist’s car continued off the road onto the landowner’s property and smashed into one of the beehives, driving a swarm of bees out of the hive. The motorcyclist, who suffered only a few bruises when his motorcycle crashed, saw the swarm of bees and started to run across the road to get away from them. He stumbled and was struck by a truck, causing him to suffer several broken bones and serious internal injuries.

Can the motorcyclist recover any damages from the landowner?

A
Yes, because the landowner is strictly liable for injury caused by the honeybees.

B
Yes, because the motorcyclist was a traveler on a public road.

C
No, because the honeybees did not directly inflict injury on the motorcyclist.

D
No, because the landowner exercised due care in her operation of the beehives.

A

CORRECT ANSWER: D.

The landowner will prevail because there is no evidence that she was negligent in her operation or maintenance of her beehives, and she is not strictly liable for the bees getting loose. Honeybees kept in a hive on a honey farm are domestic animals just like other farm animals, so strict liability does not apply. In contrast to keepers of wild animals, the owner of a domestic animal is not strictly liable for the injuries it causes. Strict liability would only apply if the owner has knowledge of that particular animal’s dangerous propensities (i.e., propensities more dangerous than normal for that species). Here, while honeybees as a class can inflict harm by stinging, there is no indication that any of these particular honeybees were more aggressive or dangerous than normal. Hence, (A) is incorrect because the landowner would not be strictly liable for the injury to the motorcyclist. (B) is incorrect because the fact that the motorcyclist is a traveler on a public road establishes only that the landowner, as the adjacent landowner, owes a duty of ordinary care as to dangerous conditions and active operations on her property. Here, the facts do not establish that the landowner acted unreasonably in the placement of her beehives. (C) is incorrect because another vehicle on the highway striking the motorcyclist was a foreseeable intervening force that did not break the causal connection between the release of the bees and the motorcyclist’s injury. If strict liability were applicable here, the harm must result from the “normally dangerous propensity” of the animal involved, but fleeing from the perceived danger is part of the risk that the dangerous propensity creates. Similarly, if the landowner were negligent, the fact that the bees did not reach the motorcyclist would not cut off the landowner’s liability to the motorcyclist.